Hesi Questions Part 4

Your page rank:

Total word count: 22036
Pages: 80

Calculate the Price

- -
275 words
Looking for Expert Opinion?
Let us have a look at your work and suggest how to improve it!
Get a Consultant

The nurse is caring for an older client with arthritis. The client has difficulty standing from and lowering into a chair because of pain. The nurse uses wooden blockers to elevate the chair legs, which helps the client sit and stand with little discomfort. Which critical thinking attitude is involved in this situation?

Humility

Curiosity

Integrity

Creativity

Creativity

The nurse is teaching the parent of an infant about inspecting the crib before putting an infant to sleep. Which statement made by the parent indicates a need for further education?

"I should remove mobiles from the infant."

"I should attach crib toys with hanging strings."

"I should check whether the crib’s mattress fits snugly."

"I should disassemble and throw away the unsafe cribs."

"I should attach crib toys with hanging strings."

While reviewing the performance of a newly appointed nurse, the chief operational officer finds that the nurse excels at using reflective journaling. What activity of the nurse would lead the chief operational officer to this conclusion?

The nurse shares constructive criticism with his or her team members.

The nurse meets with colleagues regularly to discuss work experience.

The nurse recalls, thinks, analyzes and learns from day-to-day work situations.

The nurse organizes or connects information in a way so the diverse information about a client forms meaningful patterns.

The nurse recalls, thinks, analyzes and learns from day-to-day work situations.

What are physiologic symptoms assessed in a client with sleep deprivation?

Ptosis and blurred vision

Agitation and hyperactivity

Confusion and disorientation

Increased sensitivity to pain

Decreased auditory alertness

Ptosis and blurred vision Increased sensitivity to pain

An advanced practice registered nurse (APRN) is caring for a pregnant woman. Which type of APRN would care for this client?

Clinical nurse specialist (CNS)

Certified nurse midwife (CNM)

Certified nurse practitioner (CNP)

Certified registered nurse anesthetist (CRNA)

Certified nurse midwife (CNM)

The nurse is caring for a client admitted with chronic obstructive pulmonary disease (COPD). The nurse should monitor the results of which laboratory test to evaluate the client for hypoxia?

Red blood cell count

Sputum culture

Arterial blood gas

Total hemoglobin

Arterial blood gas

A nurse is performing physical assessment of four female clients who came for a general checkup. Which client is most at risk of developing breast cancer?

Client B

Which subdimension would form a part for the caring process "doing for" according to the Swanson’s theory of caring?

Focusing

Protecting

Comforting

Seeking cues

Generating alternatives

Protecting Comforting

A nurse is assessing several clients. Which client will require parenteral nutrition?

A client with brain neoplasm

A client with anorexia nervosa

A client with inflammatory bowel disease

A client with severe malabsorption disorder

A client with severe malabsorption disorder

A client has seeds containing radium implanted in the pharyngeal area. What should the nurse include in the client’s plan of care?

Have the client void every 2 hours.

Maintain the client in an isolation room.

Spend time with the client to allow verbalization of feelings.

Wear two pairs of gloves when touching the client during care.

Maintain the client in an isolation room.

A client presents with bilateral leg pain and cramping in the lower extremities. The client has a history of cardiovascular disease, diabetes, and varicose veins. To guide the assessment of the pain and cramping, the nurse should include which question when completing the initial assessment?

"Does walking for long periods of time increase your pain?"

"Does standing without moving decrease your pain?"

"Have you had your potassium level checked recently?"

"Have you had any broken bones in your lower extremities?"

"Does walking for long periods of time increase your pain?"

A nurse finds that an older adult has reduced consciousness and fatigue and imagines something that is unreal. Which condition does the nurse suspect in the client?

Delirium

Dementia

Depression

Alzheimer’s disease

Delirium

A client admitted to the hospital with a diagnosis of malabsorption syndrome exhibits signs of tetany. The nurse concludes that the tetany was precipitated by the inadequate absorption of which electrolyte?

Sodium

Calcium

Potassium

Phosphorus

Calcium

The nurse is assessing a client with impaired hearing. Which action of the nurse is most important for establishing a good communication with the client?

Speaking at a normal volume

Reducing environmental noise

Getting the client’s attention before speaking

Rephrasing rather than repeating if misunderstood

Getting the client’s attention before speaking

The nurse who works in a birthing unit understands that newborns may have impaired thermoregulation. Which nursing interventions may help prevent heat loss in the newborns?

The nurse keeps the newborn covered in warm blankets.

The nurse keeps the newborn under the radiant warmer.

The nurse places the newborn on the mother’s abdomen.

The nurse measures the newborn’s temperature regularly.

The nurse encourages the mother to feed the newborn well to maintain the fluid balance.

The nurse keeps the newborn covered in warm blankets. The nurse keeps the newborn under the radiant warmer. The nurse places the newborn on the mother’s abdomen.

Before discharge after a myocardial infarction, a client asks the nurse when sexual activity can be resumed. What is the best response by the nurse?

"Two weeks is the usual waiting time."

"How long do you think you should wait?"

"Have you discussed this with your primary healthcare provider?""

"You should wait until your heart feels stronger."

"Have you discussed this with your primary healthcare provider?""

A visitor comes to the nursing station and tells the nurse that a client and a relative had a fight and that the client is now lying unconscious on the floor. What is the most important action the nurse needs to take?

Ask the client if he or she is okay.

Call security from the room.

Find out if there is anyone else in the room.

Ask security to make sure the room is safe.

Ask security to make sure the room is safe.

The registered nurse is teaching a nursing student about bulimia nervosa in adolescents. Which statement made by the nursing student indicates effective learning?

"The client claims to feel fat despite being underweight."

"The client experiences recurrent episodes of binge eating."

"The client exhibits intense fear of gaining weight although underweight."

"The client refuses to maintain body weight over a minimal ideal body weight."

"The client experiences recurrent episodes of binge eating."

A client experiences a muscle sprain of the ankle. When assessing the injury, the nurse discovers that a hematoma is developing, edema is present, and the client reports tenderness when the ankle is palpated. The nurse anticipates that the plan of care will include the application of what?

Binder

Ice bag

Elastic bandage

Warm compress

Ice bag

The nurse is caring for an African American client with renal failure. The client states that the illness is a punishment for sins. Which cultural health belief does the client communicate?

Yin/Yang balance

Biomedical belief

Determinism belief

Magicoreligious belief

Magicoreligious belief

When the nurse arrives at 8:00 am, a client has a 1000 mL bag of D5W hanging, with 450 mL infused during the prior shift. The IV infusion is to deliver 100 mL per hour. At 11:00 am the healthcare provider changes the prescription for the intravenous solution to 1000 mL 0.9% sodium chloride to be administered at 75 mL per hour and changes the dietary order from nothing by mouth to clear liquids. From 1:00 pm to the end of the 12-hour shift at 8:00 pm, the client has 4 oz (120 mL) of apple juice, a half cup of tea, a half cup of gelatin, and 6 oz (180 mL) of water. How many milliliters should the nurse document as the client’s total fluid intake for the 12-hour shift? Record your answer using a whole number.

1515mL **1/2 cup= 4oz

What should the nurse teach the young mother about the nutritional needs of the newborn?

The newborn should be breastfed for the first twelve months.

The newborn should be given 2% cow’s milk if breast feeding is not possible.

The newborn should receive solid food in addition to milk starting from the fourth month.

The breastfed newborn should receive iron supplements during the first four months.

The newborn should be breastfed for the first twelve months.

Which nursing intervention is most appropriate for a client in skeletal traction?

Add and remove weights as the client desires.

Assess the pin sites at least every shift and as needed.

Ensure that the knots in the rope are tied to the pulley.

Perform range of motion to joints proximal and distal to the fracture at least once a day.

Assess the pin sites at least every shift and as needed.

An older adult who is in acute care has a risk of skin breakdown. Which interventions are beneficial to the client?

Providing meticulous skin care

Reducing shear forces and friction

Providing beverages and snacks frequently

Using a support surface base all the time

Avoiding pressure with proper positioning

Providing meticulous skin care Reducing shear forces and friction Avoiding pressure with proper positioning

A client with limited mobility is being discharged. To prevent urinary stasis and formation of renal calculi, what should the nurse instruct the client to do?

Increase oral fluid intake to 2 to 3 L/day.

Maintain bed rest after discharge.

Limit fluid intake to 1 L/day.

Void at least every hour.

Increase oral fluid intake to 2 to 3 L/day.

The nurse is assessing a client following abdominal surgery. Which assessment findings should the nurse use to form a data cluster?

The client reports pain with movement.

The client has pain over the surgical area.

The client wants to know when he can go home.

The client rates the pain as 8 on a scale of 0 to 10.

The client has concerns about caring for the wound.

The client reports pain with movement. The client has pain over the surgical area. The client rates the pain as 8 on a scale of 0 to 10.

The nurse noticed the breathing rate as regular and slow while assessing a client for respiration. What could be the condition of the client?

Apnea

Bradypnea

Tachypnea

Hyperpnea

Bradypnea

While examining a client, a nurse finds a circumscribed elevation of the skin filled with serous fluid on the cheek. The lesion is 0.6 cm in diameter. What does the nurse suspect the finding to be?

Papule

Vesicle

Nodule

Pustule

Vesicle

A nurse is caring for an older adult with a hearing loss secondary to aging. What can the nurse expect to identify when assessing this client?

Dry cerumen

Tears in the tympanic membrane

Difficulty hearing high pitched voices

Decrease of hair in the auditory canal

Overgrowth of the epithelial auditory lining

Dry cerumen Difficulty hearing high pitched voices **Cerumen (ear wax) becomes drier and harder as a person ages. Generally, female voices have a higher pitch than male voices; older adults with presbycusis (hearing loss caused by the aging process) have more difficulty hearing higher pitched sounds. There is no greater incidence of tympanic tears caused by the aging process. The hair in the auditory canal increases, not decreases. The epithelium of the lining of the ear becomes thinner and drier.

What does a nurse consider the most significant influence on many clients’ perception of pain when interpreting findings from a pain assessment?

Age and sex

Physical and physiological status

Intelligence and economic status

Previous experience and cultural values

Previous experience and cultural values

The nurse is assessing a client working in a glass factory. Which occupational hazard should the nurse assess the client for?

Cataracts

Dermatitis

Lung disease

Nasopharyngeal cancer

Cataracts

While assessing the client’s skin, a nurse notices a skin condition, the pathophysiology of which involves increased visibility of oxyhemoglobin caused by an increased blood flow due to capillary dilation. Which condition is associated with this client?

Pallor

Vitiligo

Cyanosis

Erythema

Erythema

An older client is apprehensive about being hospitalized. The nurse realizes that one of the stresses of hospitalization is the unfamiliarity of the environment and activity. How can the nurse best limit the client’s stress?

Use the client’s first name.

Visit with the client frequently.

Explain what the client can expect.

Listen to what the client has to say.

Explain what the client can expect.

While assessing a client with dehydration, the nurse notices diminished skin elasticity. Which portion of the hand is used to perform this assessment?

Fingertips

Pads of fingertips

Ulnar surface of hand

Palmer surface of finger pads

Fingertips **The fingertips are used to palpate the skin for elasticity. The pads of the fingertips are used to palpate pulse amplitude. The ulnar surface of the hand is used to detect fremitus. The palmer surface of the fingertips is used to examine the thorax.

The findings of four clients who underwent eye examinations are given below.
Which client is suspected to have sustained injury to the cranial nerve III?

Client A – drooping of eyelid

A terminally ill client is furious with one of the staff nurses. The client refuses the nurse’s care and insists on doing self-care. A different nurse is assigned to care for the client. What should be the newly assigned nurse’s initial step in revising the client’s plan of care?

Get a full report from the first nurse and adjust the plan accordingly.

Ask the primary healthcare provider for a report on the client’s condition and plan appropriately.

Tell the client about the change in staff responsibilities and assess the client’s reaction.

Assess the client’s present status and include the client in a discussion of revisions to the plan of care.

Assess the client’s present status and include the client in a discussion of revisions to the plan of care.

While assessing a pediatric client, an ophthalmologist notices that the child is unable to focus on an object with both eyes simultaneously. Which other findings in the client confirms the diagnosis as strabismus?

Impaired near vision

Crossed appearance of eyes

Elevated intraocular pressure

Impaired extraocular muscles

Degeneration of central retina

Crossed appearance of eyes Impaired extraocular muscles

Which feature is most likely related to entry-level nurse competencies?

Motivating others

Setting the objectives and guiding the staff

Using a participatory approach in decision-making

Working as a team member and collaborating with other team members

Working as a team member and collaborating with other team members

A client complains of difficulty breathing. The nurse auscultates wheezing in the anterior bilateral upper lobes. What could be the possible reason for this sound?

Inflammation of the pleura

Muscular spasms in the larger airways

Sudden reinflation of groups of alveoli

High velocity airflow through an obstructed airway

High velocity airflow through an obstructed airway

A 78-year-old client who has hypertension is beginning treatment with furosemide. Considering the client’s age, what should the nurse teach the client to do?

Limit fluids at bedtime.

Change positions slowly.

Take the medication between meals.

Assess the skin for breakdown daily.

Change positions slowly.

A nurse is teaching a parenting class. What should the nurse suggest about managing the behavior of a young school-age child?

Avoid answering questions.

Give the child a list of expectations.

Be consistent about established rules.

Allow the child to plan the day’s activities.

Be consistent about established rules.

While caring for a family, the nurse finds that the family has accepted the shifts of generational roles. Which change in the family status for proceeding developmentally would the nurse observe?

Dealing with retirement

Taking on parental roles

Adjusting to a reduction in family size

Refocusing on midlife material and career issues

Dealing with retirement

The nurse is performing a skin assessment of a client. Which findings in the client may indicate a risk of skin cancer?

Lesion

Lumps

Rashes

Bruising

Dryness

Lesion Lumps Rashes

Which therapeutic communication technique is useful when the nurse and a client have a conversation and the client begins to repeat the conversation to himself or herself?

Focusing

Clarifying

Paraphrasing

Summarizing

Focusing **Focusing is a therapeutic communication technique that is useful when clients begin to repeat themselves. Clarification helps to check whether the client’s understanding is accurate by restating an unclear or ambiguous message. Paraphrasing involves restating a message more briefly using one’s own words. Summarizing is a concise review of key aspects of an interaction.

A client with a recent history of head trauma is at risk for orthostatic hypotension. Which assessment findings observed by the nurse would relate to this diagnosis?

Fainting

Headache

Weakness

Lightheadedness

Shortness of breath

Fainting Weakness Lightheadedness

A client who is in a late stage of pancreatic cancer intellectually understands the terminal nature of the illness. What are behaviors that indicate the client is emotionally accepting the impending death?

Revising the client’s will and planning a visit to a friend

Alternately crying and talking openly about death

Getting second, third, and fourth medical opinions

Refusing to follow treatments and stating they won’t help anyway

Revising the client’s will and planning a visit to a friend

A client is admitted with a suspected malignant melanoma on the arm. When performing the physical assessment, what would the nurse expect to find?

Large area of petechiae

Red birthmark that has recently become lighter in color

Brown or black mole with red, white, or blue areas

Patchy loss of skin pigmentation

Brown or black mole with red, white, or blue areas

An isolated older adult is diagnosed with cancer and fears death. Which intervention provided would help to induce relaxation and to communicate interest in the client?

Touch

Reminiscence

Reality orientation

Therapeutic communication

Touch

A client with a leg prosthesis and a history of syncopal episodes is being admitted to the hospital. When formulating the plan of care for this client, the nurse should include that the client is at risk for what?

Falls

Impaired cognition

Imbalanced nutrition

Impaired gas exchange

Falls

A client is admitted with metabolic acidosis. The nurse considers that two body systems interact with the bicarbonate buffer system to preserve healthy body fluid pH. What two body systems should the nurse assess for compensatory changes?

Skeletal and nervous

Circulatory and urinary

Respiratory and urinary

Muscular and endocrine

Respiratory and urinary

Which psychosocial health concern involves accepting descriptive statements stated by a confused older client?

Reminiscence

Reality orientation

Validation therapy

Therapeutic communication

Validation therapy

A nurse assesses a client with dry and brittle hair, flaky skin, a beefy-red tongue, and bleeding gums. The nurse recognizes that these clinical manifestations are most likely a result of what?

A food allergy

Noncompliance with medications

Side effects from medications

A nutritional deficiency

A nutritional deficiency

The nurse is assisting a client in labor. Which intervention should the nurse perform as soon as the newborn is delivered?

Remove nasopharyngeal secretions

Cover the newborn in a warm blanket

Determine the newborn’s Apgar score

Place the newborn directly on the mother’s abdomen

Remove nasopharyngeal secretions **The most important intervention immediately after the newborn is delivered is to maintain an open airway. The nurse must first remove the nasopharyngeal and oropharyngeal secretions with a suction or a bulb syringe to ensure airway patency. Newborns are easily susceptible to heat loss and cold stress. Therefore, the nurse should wrap the newborn in a warm blanket. An Apgar assessment is generally conducted between one and five minutes after birth. A healthy newborn may be placed directly on the mother’s abdomen and covered in warm blankets after a patent airway is maintained. These three interventions should be performed after the newborn’s airways are cleared.

Which nursing action is a part of the evaluation phase of the critical thinking process?

Collecting all the data in order

Looking at all the situations objectively

Support the findings and drawing conclusions

Be open-minded to information about a client

Using several criteria to determine the effectiveness of a nursing intervention

Looking at all the situations objectively Using several criteria to determine the effectiveness of a nursing intervention **During the evaluation phase of the critical thinking process, the nurse should look at the situations objectively to identify the client’s response to interventions. The results of the nursing actions should be evaluated using criteria such as expected outcomes, pain characteristics, and learning objectives. The nurse demonstrates interpretation skills by collecting all the data in order. Supporting one’s findings and drawing conclusions reflects the explanation aspect of the critical thinking process. The nurse should be open-minded while looking at the information about the client to help in accurate analysis.

Which healthcare factors create barriers that prevent older adults from participating in healthcare promotion and disease prevention?

Finance

Activity level

Transportation

Personal motivation

Previous healthcare experience

Personal motivation Previous healthcare experience

Which component of decision-making refers to the duties and activities an individual is employed to perform?

Authority

Autonomy

Responsibility

Accountability

Responsibility

The registered nurse is teaching a nursing student about how to educate clients based on their developmental capacity. Which statements made by the nursing student are applicable for older adults?

"I should encourage independent learning."

"I should keep the teaching sessions short."

"I should involve the client in any discussion or activity."

"I should encourage learning through pictures and short stories."

"I should teach the client psychomotor skills to maintain his or her health."

"I should keep the teaching sessions short." "I should involve the client in any discussion or activity."

The nurse recognizes that which is the mental process most sensitive to deterioration with aging?

Judgment

Intelligence

Creative thinking

Short-term memory

Short-term memory

While assessing a neonate’s temperature, the nurse observes a drop in the body temperature. What is the most appropriate reason for this temperature drop?

Decreased nonshivering thermogenesis

Which questions should the nurse ask the client when obtaining the health history?

"Tell me about your food habits."

"Do you use alcohol or tobacco?"

"Have you sustained any personal loss recently?"

"Have you ever experienced any allergic reactions?"

"Does any family member have a long-term illness?"

"Tell me about your food habits." "Do you use alcohol or tobacco?" "Have you ever experienced any allergic reactions?"

The nurse assessed a client’s pulse rate and recorded the score as 3+. What is the strength of the pulse?

Strong

Bounding

Expected

Diminished

Strong

A registered nurse (RN) is performing a physical assessment of four clients with various medical conditions as shown in the chart. Which client is expected to have concavely curved nails?

iron deficiency anemia

Which degree of edema will result in a 6-mm deep indentation upon pressure application?

4+

3+

2+

1+

3+

While assessing the pupils of a client, a healthcare professional notices pupillary dilatation. Which drug intake might have resulted in this condition?

Heroin

Atropine

Morphine

Pilocarpine

Atropine

A nursing student is recording the radial pulse rate in a client with dysrhythmias and documented a radial pulse of 80 beats per minute. The registered nurse reassesses the client and notices a pulse deficit of 15. What is the client’s apical pulse?

95

85

75

65

95

Which statement best describes a diagnostic label?

It is a condition that responds to nursing interventions.

It describes the essence of the client’s response to health conditions.

It describes the characteristics of the client’s response to health conditions.

It is identified from the client’s assessment data and associated with the diagnosis.

It describes the essence of the client’s response to health conditions.

Which landmark is correct for a nurse to use when auscultating the mitral valve?

Left fifth intercostal space, midaxillary line

Left fifth intercostal space, midclavicular line

Left second intercostal space, sternal border

Left fifth intercostal space, sternal border

Left fifth intercostal space, midclavicular line

Which term refers to a blowing sound created by turbulence caused by narrowing of arteries while assessing for carotid pulse?

Bruit

Ectropion

Entropion

Borborygmi

Bruit

The nurse cares for an unconscious client who underwent head surgery. Which site would be best used to monitor body temperature?

Skin

Oral

Axilla

Rectal

Rectal

A registered nurse is teaching a nursing student about skin assessment. Which statement made by the nursing student indicates the need for further teaching?

"Skin assessments are best performed in daylight."

"Skin assessments performed at cool room temperatures can result in cyanosis."

"Skin assessment performed at warm room temperatures can result in vasodilatation."

"In the absence of sunlight, skin assessments are performed best with other sources of light instead of fluorescent light."

"In the absence of sunlight, skin assessments are performed best with other sources of light instead of fluorescent light."

When monitoring fluids and electrolytes, the nurse recalls that the major cation-regulating intracellular osmolarity is what?

Sodium

Potassium

Calcium

Calcitonin

Potassium

What is the priority nursing intervention for a client during the immediate postoperative period?

Monitoring vital signs

Observing for hemorrhage

Maintaining a patent airway

Recording the intake and output

Maintaining a patent airway

A nurse is assessing an older adult client. Which clinical findings are expected responses to the aging process?

Slowed neurologic responses

Lowered intelligence quotient

Long-term memory impairment

Forgetfulness about recent events

Reduced ability to maintain an erection

Slowed neurologic responses Forgetfulness about recent events Reduced ability to maintain an erection

A client is admitted for surgery. Although not physically distressed, the client appears apprehensive and withdrawn. What is the nurse’s best action?

Orient the client to the unit environment.

Have a copy of hospital regulations available.

Explain that there is no reason to be concerned.

Reassure the client that the staff is available if the client has questions.

Orient the client to the unit environment.

What should the nurse teach the parents of an infant about the use of car seats?

The infant should ride in a front-facing car safety seat.

The infant should ride in a car safety seat until one year of age.

The infant should be restrained properly in a federally approved car safety seat.

The infant should always ride in a car seat restrained to the front seat of the car.

The infant should be restrained properly in a federally approved car safety seat.

The nurse caring for a client with a systemic infection is aware that the assessment finding that is most indicative of a systemic infection is what?

White blood cell (WBC) count of 8200/mm 3 (8.2 X 10 9/L)

Bilateral 3+ pitting pedal edema

Oral temperature of 101.3° F (38.5° C)

Pale skin and nail beds

An elevated temperature of 101.3° F (38.5° C) is most indicative of a systemic infection. A WBC count of 8200/mm 3 (8.2 X 10 9/L) is within the normal range of 5000 to 10,000/mm 3 (5 to 10 X 10 9/L). Pedal edema is generally not related to an infectious process. Pale skin and nail beds may be related to an infectious process, but not necessarily.

A client being treated for influenza A (H1N1) is scheduled for a computed tomography (CT) scan. To ensure client and visitor safety during transport, the nurse should take which precaution?

Place a surgical mask on the client.

Other than standard precautions, no additional precautions are needed.

Minimize close physical contact.

Cover the client’s legs with a blanket.

Place a surgical mask on the client.

A nurse is caring for a client who has paraplegia as a result of a spinal cord injury. Which rehabilitation plan will be most effective for this client?

Arrangements will be made by the client and the client’s family.

The plan is formulated and implemented early in the client’s care.

The rehabilitation is minimal and short term, because the client will return to former activities.

Arrangements will be made for long-term care, because the client is no longer capable of self-care.

The plan is formulated and implemented early in the client’s care.

According to Swanson’s caring process, the nurse must know the client. Which factors enable the nurse to know the client better?

Economic constraints

Continuity of care by the nursing staff

Fewer nurses in the healthcare facility

Collection of data about the client’s clinical condition

Engagement in a caring relationship without assumptions

Continuity of care by the nursing staff Collection of data about the client’s clinical condition Engagement in a caring relationship without assumptions

A client is admitted to the hospital with severe diarrhea, abdominal cramps, and vomiting after eating. These symptoms have lasted 5 days. Upon further assessment, the primary healthcare provider finds that the symptoms occurred after the client ate eggs, salad dressings, and sandwich fillings. Which food borne disease would be suspected in this client?

Listeriosis

Shigellosis

Salmonellosis

Staphylococcus

Salmonellosis

While assessing an elderly client, a nurse infers cognitive impairment. Which statements made by the client confirm the nurse’s conclusion?

"I have difficulty judging things."

"I forget to take medicines."

"I am unable to do financial calculations."

"I get confused about the proper date and time."

"I am unable to recall words during conversations with my family."

"I have difficulty judging things." "I am unable to do financial calculations." "I am unable to recall words during conversations with my family." **Poor judgment, loss of the ability to calculate, and loss of language skills are related to cognitive impairment.

A client has a pressure ulcer that is full thickness with necrosis into the subcutaneous tissue down to the underlying fascia. The nurse should document the assessment finding as which stage of pressure ulcer?

Stage I

Stage II

Stage III

Unstageable

A pressure ulcer with necrotic tissue is unstageable. The necrotic tissue must be removed before the wound can be staged.

What are the goals of care when working with families according to the family health system?

To improve family health or well-being

To help the family prepare for later transitions

To assist in family management of illness conditions

To promote positive family behaviors to achieve essential tasks

To achieve health outcomes related to the family’s areas of concern

To improve family health or well-being To assist in family management of illness conditions To achieve health outcomes related to the family’s areas of concern

A client complains to the nurse that a staff member did not respond to the client’s call. The nurse politely reassures the client, and makes the client comfortable. The nurse speaks to the staff member about the incident and solves the problem. Which critical thinking attitude has the nurse demonstrated in this situation?

Fairness

Discipline

Confidence

Responsibility

Fairness

The nurse is caring for a nonambulatory client with a reddened sacrum that is unrelieved by repositioning. What nursing diagnosis should be included in the client’s plan of care?

Risk for pressure ulcer

Risk for impaired skin integrity

Impaired skin integrity, related to infrequent turning and repositioning

Impaired skin integrity, related to the effects of pressure and shearing force

Impaired skin integrity, related to the effects of pressure and shearing force

A 58-year-old client is planning to retire. Which action would be appropriate in this situation?

Assessing the activity level

Assessing issues related to income

Assessing the family to conduct an environmental check

Assessing the options of public transportation and number of community activities

Assessing issues related to income

The nurse is caring for a client before, during, and immediately after surgery. Which type of care is provided to the client?

Care that supports physical functioning

Care that supports homeostatic regulation

Care that supports psychosocial functioning

Care that provides immediate short-term help in physiological crises

Care that supports homeostatic regulation

The nurse is providing postprocedure care for a client who had a liver biopsy. To prevent hemorrhage, it is the nurse’s highest priority to place the client in what position?

Prone

High Fowler

On the right side

Trendelenburg

On the right side **Placing a client on the right side after a liver biopsy compresses the liver against the abdominal wall, thus holding pressure on the biopsy site and allowing clot formation. There is no indication that the prone, high Fowler, and Trendelenburg positions are beneficial or appropriate for the client.

What type of interview is most appropriate when a nurse admits a client to a clinic?

Directive

Exploratory

Problem solving

Information giving

Directive

While caring for a client dealing with pain, the nurse assesses the health status and prioritizes his or her needs. Which phase of the helping relationship is observed?

Working phase

Orientation phase

Termination phase

Preinteraction phase

Orientation phase

A nurse is caring for an elderly client with dementia who has developed dehydration as a result of vomiting and diarrhea. Which assessment best reflects the fluid balance of this client?

Skin turgor

Intake and output results

Client’s report about fluid intake

Blood lab results

Blood lab results

The nurse asks the client to shrug the shoulders and to turn the head against passive resistance. Which cranial nerve is involved in this action?

Cranial nerve II

Cranial nerve XI

Cranial nerve VI

Cranial nerve VII

Cranial nerve XI

Which client is suspected to have an increased risk of hyperlipidemia?

Client with corneal arcus

Client with periorbital edema

Client with decreased skin turgor

Client with paleness of conjunctivae

Client with yellow lipid lesions on eyelids

Client with corneal arcus Client with yellow lipid lesions on eyelids

A client who is scheduled for a surgical resection of the colon and creation of a colostomy for a bowel malignancy asks why preoperative antibiotics have been prescribed. The nurse explains that the primary purpose is to do what?

Decrease peristalsis

Minimize electrolyte imbalance

Decrease bacteria in the intestines

Treat inflammation caused by the malignancy

Decrease bacteria in the intestines

The nurse is performing a breast assessment. Which statement made by the client indicates the risk of breast cancer?

"I had a late onset of menarche."

"My first child was born when I was 32."

"I noticed a slight discharge from a nipple."

"I perform breast self-examinations frequently."

"I consume two to four glasses of alcohol a day."

"My first child was born when I was 32." "I noticed a slight discharge from a nipple." "I consume two to four glasses of alcohol a day."

Health promotion efforts for a chronically ill client should include interventions related to primary prevention. What should this include?

Encouraging daily physical exercise

Performing yearly physical examinations

Providing hypertension screening programs

Teaching a person with diabetes how to prevent complications

Encouraging daily physical exercise

The registered nurse teaches a nursing student about the implementation process of nursing. Which example does the registered nurse use while describing indirect care interventions using his or her knowledge?

The nurse counseling a client at the time of grief

The nurse administering an intravenous infusion to a client

The nurse teaching the client about an appropriate nutrition plan

The management of the client’s environment to prevent infections

The management of the client’s environment to prevent infections

A primary healthcare provider tells a client about the diagnosis of inoperable cancer and that the client does not have long to live. After the primary healthcare provider leaves, the client says to the nurse, "I feel fine. I probably only have the flu." The nurse determines that the client is in the denial stage of grief. What should the nurse do to help meet the client’s emotional needs?

Reassure the client that everything will be alright.

Leave the client alone to confront feelings of impending loss.

Encourage the denial until the client is able to accept reality.

Allow the denial and be available to discuss the situation with the client.

Allow the denial and be available to discuss the situation with the client.

A client reports to the hospital with skin lesions. Upon physical examination, the nurse notices circumscribed elevations of the skin, measuring about 0.5 × 0.5 cm. The lesions are filled with serous fluid. What is the suspected cause of these skin lesions?

Venous stasis ulcer

Arterial insufficiency

Staphylococcal infection

Herpes simplex infection

Herpes simplex infection

A nurse is assessing four different clients. Which client is at risk of pleuritis?

Client A

Client B

Client C

Client D

Client D- pleural friction rub over anterior lateral lung field

A nurse is assessing a client with a history of marijuana use. Which long-term effects are associated with marijuana?

Lung cancer

Emphysema

Heart disease

Laryngeal disorder

Stroke

Chronic nasal irritation

Lung cancer Emphysema Heart disease

A registered nurse notices that a student nurse who is assessing the blood pressure in a client is deflating the cuff too rapidly. What is the probable reading of blood pressure that the student nurse could have obtained if the actual blood pressure of the client is 140/90 mm Hg?

130/80 mm Hg

150/100 mm Hg

140/100 mm Hg

130/100 mm Hg

130/100 mm Hg

A registered nurse is teaching a nursing student how to assess for edema. Which statement made by the student indicates the need for further education?

"Edema results in the separation of skin from pigmented and vascular tissue."

"Pitting edema leaves an indentation on the site of application of pressure."

"Trauma or impaired venous return should be suspected in clients with edema."

"If the pressure on an edematous site leaves an indentation of 2 mm, a score of 2+ is given."

"If the pressure on an edematous site leaves an indentation of 2 mm, a score of 2+ is given."

A client suffers hypoxia and a resultant increase in deoxygenated hemoglobin in the blood. What are the best sites to assess this condition?

Lips

Sclera

Mouth

Sacrum

Nail beds

Shoulders

Lips Mouth Nail Beds

A client is admitted to the hospital after an accident. The nurse uses the Glasgow Coma Scale (GCS) with the client. The client is alert and opens his or her eyes when there is a sound or when someone talks. When questions are asked, the client answers in a confused manner. The client obeys commands, such as being asked to move a leg. What would be the client’s total score? Record your answer using a whole number.

13

While performing a physical assessment of a female client, the nurse positions the client in Sims’ position. Which body system will be assessed in this position?

Heart

Vagina

Rectum

Female genitalia

Musculoskeletal system

Vagina Rectum

A 78-year-old client who has hypertension is beginning treatment with furosemide. Considering the client’s age, what should the nurse teach the client to do?

Limit fluids at bedtime.

Change positions slowly.

Take the medication between meals.

Assess the skin for breakdown daily.

Change positions slowly.

A nurse is caring for a client admitted with cardiovascular disease. During the assessment of the client’s lower extremities, the nurse notes that the client has thin, shiny skin; decreased hair growth; and thickened toenails. What might this indicate?

Venous insufficiency

Arterial insufficiency

Phlebitis

Lymphedema

Arterial insufficiency

A nurse is assessing four clients.
Which client is at the highest medical risk of coronary heart disease and hypertension?

Client D – 145 cm and 67 kg

A client shows an increase in rate respirations that are abnormally deep and regular. What condition would the nurse expect?

Hypoventilation

Biot’s respiration

Kussmaul’s respiration

Cheyne-Stokes respiration

Kussmaul’s respiration

When assessing a client, the nurse auscultates a murmur at the second left intercostal space (ICS) along the sternal border. This reflects sound from which valve?

Aortic

Mitral

Pulmonic

Tricuspid

Pulmonic

The unlicensed assistive personnel (UAP) recorded the vital signs of four clients. Which client needs immediate nursing interventions?

Client A

Which position is indicated to assess the musculoskeletal system and is contraindicated in clients with respiratory difficulties?

Sims position

Prone position

Supine position

Knee-chest position

Prone position

A nurse must establish and maintain an airway in a client who has experienced a near-drowning in the ocean. For which potential danger should the nurse assess the client?

Alkalosis

Renal failure

Hypervolemia

Pulmonary edema

Pulmonary edema

Which pulse site is used for the Allen’s test?

Ulnar

Popliteal

Brachial

Femoral

Ulnar

The nurse tells a client undergoing diuretic therapy to avoid working in the garden on hot summer days. What condition is the nurse trying to prevent in this client?

Frostbite

Heatstroke

Hypothermia

Hyperthermia

Heatstroke

A nurse is caring for a client who underwent cardiac catheterization. The client’s skin was found to be blanched, and there was formation of edema of 15.2 cm (1-6 inches) at the site of catheterization. Upon further assessment, the skin was found to be cool, and the client complains of tenderness. Which condition does the nurse expect?

Phlebitis

Infection

Infiltration

Circulatory overload

Infiltration

Which activity by the community nurse can be considered an illness prevention strategy?

Encouraging the client to exercise daily

Arranging an immunization program for chicken pox

Teaching the community about stress management

Teaching the client about maintaining a nutritious diet

Arranging an immunization program for chicken pox

After performing an optical assessment on a client, a primary healthcare provider notices impaired near vision. Which other finding in this client confirms the diagnosis as presbyopia?

Loss of elasticity of the lens

Increased opacity of the lens

Elevated intraocular pressure

Noninflammatory changes in eyes

Loss of elasticity of the lens

An older adult with chills arrived to hospital. The nurse assesses the client’s vital signs and determined the client has a fever. What would be the client’s rectal temperature?

36.0ºC

36.8ºC

37.2ºC

38.5ºC

38.5ºC

Nurses care for clients in a variety of age groups. In which age group is the occurrence of chronic illness the greatest?

Older adults

Adolescents

Young children

Middle-aged adults

Older adults

While performing a neck assessment, the nurse finds the client has enlarged lymph nodes. The client also had a history of intravenous drug use and bisexual activity. What would be the possible diagnosis?

Cancer

Thyroid disease

Tracheal displacement

Human immunodeficiency virus (HIV) infection

Human immunodeficiency virus (HIV) infection

A client with cystic fibrosis asks why the percussion procedure is being performed. The nurse explains that the primary purpose of percussion is to do what?

Relieve bronchial spasms

Increase depth of respirations

Loosen pulmonary secretions

Expel carbon dioxide from the lungs

Loosen pulmonary secretions

What should the nurse do when the defining characteristics of assessment data for a client can apply to more than one diagnosis?

Reassess the client.

Reject all diagnoses.

Gather more information.

Identify related factors.

Review all defining characteristics.

Gather more information. Identify related factors. Review all defining characteristics.

When trying to promote effective learning in a client with a newly diagnosed disease, what should the nurse consider?

Client’s past experiences

Client’s personal resources

Stress of the total situation

Type of onset of the disease

Client’s past experiences

A client who underwent surgery feels pain in the lower abdomen. The nurse provides pain relief but the client is still reporting pain. Which actions of the nurse would help the client to get relief? .

Learning more about the client

Looking for different distraction techniques

Using known scientific and practice-based criteria

Involving the client’s family in creating a new plan for pain relief

Bringing co-workers together to find a solution

Looking for different distraction techniques Involving the client’s family in creating a new plan for pain relief

The nurse finds that the client’s fever spikes and falls without a return to a normal level. Which pattern of fever is this a characteristic of?

Relapsing

Sustained

Remittent

Intermittent

Remittent

A nurse explains to an obese client that the rapid weight loss during the first week after initiating a diet is because of fluid loss. The weight of extracellular body fluid is approximately 20% of the total body weight of an average individual. Which component of the extracellular fluid contributes the greatest proportion to this amount?

Plasma

Interstitial

Dense tissue

Body secretions

Interstitial

Following a surgery on the neck, the client asks the nurse why the head of the bed is up so high. The nurse should tell the client that the high-Fowler position is preferred for what reason?

To avoid strain on the incision

To promote drainage of the wound

To provide stimulation for the client

To reduce edema at the operative site

To reduce edema at the operative site

After assessing the muscle functionality of a client, the nurse assigns a grade of F (fair) on the Lovett scale in the client. What is the muscle functionality of the client?

Full range of motion with gravity

Full range of motion with gravity eliminated

Full range of motion against gravity with full resistance

Full range of motion against gravity with some resistance

Full range of motion with gravity

The nurse who is working during the 8:00 am to 4:00 pm shift must document a client’s fluid intake and output. An intravenous drip is infusing at 50 mL per hour. The client drinks 4 oz of orange juice and 6 oz of tea at 8:30 am and vomits 200 mL at 9:00 am. At 10:00 am the client drinks 60 mL of water with medications; the client voids 550 mL of urine at 11:00 am. At 12:30 pm, 3 oz of soup and 4 oz of ice cream are ingested. The client voids 450 mL at 2:00 pm. Calculate the total intake for the 8:00 am to 4:00 pm shift. Record your answer using a whole number.

mL

970 ml 1 oz = 30ml

A nurse is assessing a client’s nails and finds a slight convex curve at the angle from the skin to nail base of about 160 degrees. Which condition does the nurse suspect?

Clubbing

Paronychia

Koilonychia

Normal finding

Normal finding

A client’s breath has a sweet, fruity odor. Which condition is likely affecting this client?

Gum disease

Uremic acidosis

Diabetic acidosis

Infection inside a cast

Diabetic acidosis

The nurse is performing an assessment of the client’s reproductive system. Which finding of the past medical history indicates the client is at risk of cervical cancer?

Vaginal discharge

Ovarian dysfunction

Human papilloma virus infection

Hematuria and urinary incontinence

Human papilloma virus infection

The nurse is assessing a client who reports breathlessness. Which activity best ensures that the nurse obtains accurate and complete data to prevent a nursing diagnostic error?

Assess the client’s lungs.

Assess the client for pain.

Obtain details of smoking habits.

Ask about the onset of breathlessness.

Assess the client’s lungs.

What is a nurse’s most appropriate response, based on current research, when asked about spanking as a disciplinary technique?

"Effectiveness depends on the child’s age."

"Spanking is strongly suggestive of negative role behavior."

"Spanking may be the only option when no other technique works."

"Research studies have shown it to be an effective disciplinary technique."

"Spanking is strongly suggestive of negative role behavior."

A primary healthcare provider prescribes a urinalysis for a client with an indwelling catheter. To ensure that an appropriate specimen is obtained, the nurse would obtain the specimen from which site?

Tubing injection port

Distal end of the tubing

Urinary drainage bag

Catheter insertion site

Tubing injection port

The registered nurse is teaching a nursing student about ways to minimize heat radiation. Which statements made by the nursing student indicate effective learning?

"I will apply an ice pack to the client."

"I will cover the client with dark clothes."

"I will instruct the client to remove extra clothes."

"I will instruct the client to lie in the fetal position."

"I will advise the client to wear sparsely woven clothes."

"I will apply an ice pack to the client." "I will cover the client with dark clothes." "I will instruct the client to remove extra clothes."

What should the nurse assess to determine whether a 75-year-old individual is meeting the developmental tasks associated with aging?

Achievement of a personal philosophy

Adaptation to the children leaving home

Attainment of a sense of worth as a person

Adjustment to life in an assisted-living facility

Attainment of a sense of worth as a person

While assessing a client with chills and fever, the nurse observes that the febrile episodes are followed by normal temperatures and that the episodes are longer than 24 hours. Which fever pattern does the nurse anticipate?

Relapsing

Sustained

Remittent

Intermittent

Relapsing

What is the inflammation of the skin at the base of the nail called?

Paronychia

Koilonychia

Beau’s lines

Splinter hemorrhage

Paronychia

A registered nurse (RN) is performing a physical examination of a client with chronic obstructive pulmonary disease. Which abnormal nail bed patterns can be expected in this client?

Spoon-shaped nails

Transverse depressions in nails

Softening of nail beds and flat nails

Red or brown linear streaks in nail bed

Softening of nail beds and flat nails

Which response by the nurse during a client interview is an example of back channeling?

"All right, go on…"

"What else is bothering you?"

"Tell me what brought you here."

"How would you rate your pain on a scale of 0 to 10?"

"All right, go on…"

While assessing a client, the nurse finds inflammation of the skin at the bases of the client’s nails. What might be the reason behind this condition?

Trauma

Trichinosis

Pulmonary disease

Iron-deficiency anemia

Trauma

The nurse pulls up on the client’s skin and releases it to determine whether the skin returns immediately to its original position. What is the nurse assessing for?

Pain tolerance

Skin turgor

Ecchymosis formation

Tissue mass

Skin turgor

Which anatomic area is palpated if the nurse suspects aortic abnormalities?

D

A 16-year-old client has a blood pressure reading of 119/75. What is the approximate pulse pressure? Record your answer using a whole number __________ mm Hg

44

While assessing a client’s vascular system, the nurse finds that pulse strength is diminished or barely palpable. Which documentation is appropriate in this situation?

1+

2+

3+

4+

1+

A client with a history of hypothyroidism reports giddiness, excessive thirst, and nausea. Which parameter assessed by the nurse confirms the diagnosis as heat stroke?

Increased heart rate

Increased blood pressure

Decreased respiratory rate

Increased circulatory damage

Increased heart rate

A nurse is teaching a male client about measures to maintain sexual health and prevent transmission of sexually transmitted infections (STI). Which statement of the client indicates effective learning?

"I will use condoms when having sex with an infected partner."

"I will perform a genital self-examination every month before bathing."

"I will refrain from getting the human papilloma virus vaccine (HPV) before the age of 27 years."

"I will consult with my primary healthcare provider when there is a rash or ulcer on my genitalia."

"I will consult with my primary healthcare provider when there is a rash or ulcer on my genitalia."

Which feature is characteristic of a risk nursing diagnosis?

The diagnosis does not have related factors.

The diagnosis can be used in any health state.

The defining characteristics support the diagnostic judgment.

The defining characteristics are supported by a client’s readiness.

The diagnosis does not have related factors.

A nurse is palpating the peripheral pulse of different clients. Which client has an unacceptable heart rate?

Client 3 – 2 years, 148 bpm

Which site should be monitored for a pulse to assess the status of circulation to the foot?

Carotid artery

Femoral artery

Popliteal artery

Dorsalis pedis artery

Posterior tibial artery

Dorsalis pedis artery Posterior tibial artery

A nurse is teaching a client about measures to promote health. Which statements made by the client indicate effective learning?

"I will assess my own pulse rate after exercising."

"I will follow my hypertension treatment plan consistently."

"I will recalibrate my aneroid sphygmomanometer once a year."

"I will perform a self-assessment of my heart rate using the carotid pulse."

"I will ask my caretaker to check my blood pressure at a different time every day."

"I will assess my own pulse rate after exercising." "I will follow my hypertension treatment plan consistently." "I will perform a self-assessment of my heart rate using the carotid pulse."

A client with a head injury underwent a physical examination. The nurse observes that the client’s temperature assessments do not correspond with the client’s condition. An injury to which part of the brain may be the reason for this condition?

Pons

Medulla

Thalamus

Hypothalamus

Hypothalamus

A client reports vomiting and diarrhea for 3 days. Which clinical indicator is most commonly used to determine whether the client has a fluid deficit?

Presence of dry skin

Loss of body weight

Decrease in blood pressure

Altered general appearance

Loss of body weight

While performing a physical assessment in a client, the registered nurse (RN) notices reddish linear streaks in the nail bed. Which systemic condition can the registered nurse (RN) suspect in the client based on these assessment findings?

Syphilis

Iron deficiency anemia

Subacute bacterial endocarditis

Chronic obstructive pulmonary disease

Subacute bacterial endocarditis

During an assessment, the client complains of tenderness when the nurse palpates the calf muscle. What would be the nurse’s next assessment?

To evaluate for any reduced hair growth

To evaluate for swelling, warmth and muscle firmness

To assess for any history of ulcer formation around the calf muscle

To evaluate for venous distension in the posterolateral part of ankle

To evaluate for swelling, warmth and muscle firmness

While assessing an older adult during a regular health checkup, a nurse finds signs of elder abuse. Which physical findings would further confirm the nurse’s suspicion?

Presence of hyoid bone damage

Presence of cognitive impairment

Presence of burns from cigarettes

Presence of bed sores.

Presence of unexplained bruises on the wrist(s)

Presence of burns from cigarettes Presence of bed sores. Presence of unexplained bruises on the wrist(s)

While assessing a client, the nurse finds bluish coloration of the skin. The nurse finds that this discoloration is due to cyanosis. Which condition may be suspected?

Anemia

Liver disease

Heart disease

Autoimmune disease

Heart disease

The nurse is assessing a client who had knee replacement surgery. Which assessment finding gathered by the nurse is an example of subjective data?

The client weighs 151 lbs (68.5 Kg).

The client’s pain is 7 on a scale of 1 to 10.

The client’s fasting blood sugar is 95 mg/dL.

The client’s blood pressure is 140/90 mm/Hg.

The client’s pain is 7 on a scale of 1 to 10.

Which statement is true for collaborative problems in a client receiving healthcare?

They are the identification of a disease condition.

They include problems treated primarily by nurses.

They are identified by the primary healthcare provider.

They are identified by the nurse during the nursing diagnosis stage.

They are identified by the nurse during the nursing diagnosis stage.

A staff nurse on a medical-surgical unit has been assigned to care for a number of clients. The nurse decides to review their individual records before client contact. Which phase of the nurse-client relationship does this represent?

Working phase

Orientation phase

Termination phase

Preinteraction phase

Preinteraction phase

The nurse is performing a weight assessment for different people in a community. Which question should the nurse ask a client to determine a disease-related change in weight?

Do you follow a strict calorie intake?

Have you notices any changes in the social aspects of eating?

Are you taking diuretics or insulin?

Have you noticed any unintentional weight loss in the past six months?

Have you noticed any unintentional weight loss in the past six months?

What principal components are associated with a nurse’s time management skill?

Autonomy

Goal setting

Priority setting

Interruption control

Right communication

Goal setting Priority setting Interruption control

The registered nurse is teaching a nursing student about caring for a client who has difficulty speaking English. Which statement made by the nursing student would cause communication problems with the client?

"I will give the client a call bell."

"I will involve the client’s family members as interpreters."

"I will provide a dictionary to the client if the client can read."

"I will use boards and pictures to communication with the client."

"I will involve the client’s family members as interpreters."

Which statement of the nurse at the time of discharge would reflect the decision-making skill called autonomy?

"I accept the task of providing a discharge teaching plan."

"I understand my task of preparing a discharge teaching plan."

"I may independently develop and implement a discharge teaching plan."

"I will consult with other team members to find out why the discharge teaching plan is delayed."

"I may independently develop and implement a discharge teaching plan."

Which situation is an example of a protective touch that may elicit negative feelings in a client?

The nurse provides a back massage to a client.

The nurse obtains a blood sample for laboratory tests.

The nurse holds a client while assisting with ambulation.

The nurse withdraws from a client to escape the situation.

The nurse withdraws from a client to escape the situation.

What would be the behavioral characteristic of a slow-to-warm up child according to the theory related to temperament?

Highly active

Irritable and irregular in habits

Negative reaction to new stimuli

A positive mild-to-moderately intense mood

Negative reaction to new stimuli

After being medicated for anxiety, a client says to a nurse, "I guess you are too busy to stay with me." How should the nurse respond?

"I’m so sorry, but I need to see other clients."

"I have to go now, but I will come back in 10 minutes."

"You’ll be able to rest after the medicine starts working."

"You’ll feel better after I’ve made you more comfortable."

"I have to go now, but I will come back in 10 minutes."

A nurse is taking care of a client who has chronic back pain as a result of a work injury. What nursing considerations should be made when determining the client’s plan of care?

Ask the client about the acceptable level of pain.

Eliminate all activities that precipitate the pain.

Administer the pain medications regularly around the clock.

Use a different pain scale each time to promote patient education.

Assess the client’s pain every 15 minutes.

Ask the client about the acceptable level of pain. Administer the pain medications regularly around the clock.

A recent immigrant from mainland China is critically ill and dying. What question should the nurse ask when collecting information to meet the emotional needs of this client?

"Do you like living in this country?"

"When did you come to this country?"

"Is there a family member who can translate for you?"

"Which family member do you prefer to receive information?"

"Which family member do you prefer to receive information?"

A client with hyperthyroidism has been treated with radioactive iodine ( 131I) to destroy overactive thyroid gland cells. To reduce radiation exposure, the nurse’s principles for providing care should be based on what?

Wearing a lead-shield apron at all times

Limiting distance and time spent with the client

Wearing a radiation meter to measure exposure

Remaining at least 6 feet (1.8 m) away from the client at all times

Limiting distance and time spent with the client

Elbow restraints have been prescribed for a confused client to keep the client from pulling out a nasogastric tube and indwelling urinary retention catheter. What is most important for the nurse to do?

Have the prescription renewed every 48 hours.

Assess the client’s condition every hour.

Provide range of motion to the client’s elbows every shift.

Document output from the tube and catheter every 2 hours.

Assess the client’s condition every hour.

How can a nurse best evaluate the effectiveness of communication with a client?

Client feedback

Medical assessments

Health care team conferences

Client’s physiologic responses

Client feedback

The nurse is communicating with an older adult who has a hearing disability. Which intervention by the nurse is beneficial to promote communication?

Giving the client a chance to speak

Assuming the client is being uncooperative

Chewing gum while talking to the client

Making sure that the client knows you are speaking

Keeping the communication concise

Giving the client a chance to speak Making sure that the client knows you are speaking Keeping the communication concise

The nurse is teaching a client about adequate hand hygiene. What component of hand washing should the nurse include that is most important for removing microorganisms?

Soap

Time

Water

Friction

Friction

A client with coronary artery disease has a sudden episode of cyanosis and a change in respirations. The nurse starts oxygen administration immediately. Legally, should the nurse have administered the oxygen?

The oxygen had not been prescribed and therefore should not have been administered.

The symptoms were too vague for the nurse to determine a need for administering oxygen.

The nurse’s observations were sufficient, and therefore oxygen should have been administered.

The primary healthcare provider should have been called for a prescription before the nurse administered the oxygen.

The nurse’s observations were sufficient, and therefore oxygen should have been administered.

A primary healthcare provider tells a client about the diagnosis of inoperable cancer and that the client does not have long to live. After the primary healthcare provider leaves, the client says to the nurse, "I feel fine. I probably only have the flu." The nurse determines that the client is in the denial stage of grief. What should the nurse do to help meet the client’s emotional needs?

Reassure the client that everything will be alright.

Leave the client alone to confront feelings of impending loss.

Encourage the denial until the client is able to accept reality.

Allow the denial and be available to discuss the situation with the client.

Allow the denial and be available to discuss the situation with the client.

Which fine-motor skills may be observed in an 8 to 10 month-old infant?

Using pincer grasp well

Picking up small objects

Showing hand preference

Crawling on hands and knees

Pulling oneself to standing or sitting

Using pincer grasp well Picking up small objects Showing hand preference

A client who wakes up after a surgery spits out the oral airway placed during the recovery from anesthesia. What does this behavior indicate to the nurse?

The client’s gag reflex has returned.

The client is confused due to anesthesia.

The client is nauseated and wants to vomit.

The client’s airway is becoming obstructed.

The client’s gag reflex has returned.

On the third postoperative day after a below-the-knee amputation, a client is refusing to eat, talk, or perform any rehabilitative activities. What is the best initial approach that the nurse should take when interacting with this client?

Explain why there is a need to increase activity.

Emphasize that with a prosthesis, there will be a return to the previous lifestyle.

Appear cheerful and noncritical regardless of the client’s response to attempts at intervention.

Acknowledge that the client’s withdrawal is an expected and necessary part of initial grieving.

Acknowledge that the client’s withdrawal is an expected and necessary part of initial grieving.

What should the community nurse teach about the risk of adolescent pregnancy?

Risk for premature birth

Risk for having a large baby

Risk for chromosomal defects

Risk for increased weight gain

Risk for premature birth

Which approach is a comforting approach that communicates concern and support?

Touch

Listening

Knowing the client

Providing a positive presence

Touch

The nurse is preparing to perform endotracheal suctioning of a client with respiratory difficulties. Before beginning the procedure, what should the nurse do?

Ask the client to take several deep breaths.

Instruct the client to cough before suctioning.

Administer 100% oxygen to the client.

Change the suctioning equipment to ensure sterility.

Administer 100% oxygen to the client.

Which activity would the nurse explain can be performed by infants of aged 6 to 8 months?

Holding a pencil

Showing hand preference

Placing objects into containers

Transferring objects from hand to hand

Transferring objects from hand to hand

A client with a history of chronic obstructive pulmonary disease (COPD) is admitted with acute bronchopneumonia. The client is in moderate respiratory distress. The nurse should place the client in what position to enhance comfort?

Side-lying with head elevated 45 degrees

Sims with head elevated 90 degrees

Semi-Fowler with legs elevated

High Fowler using the bedside table as an arm rest

High Fowler using the bedside table as an arm rest

Which workers would the nurse consider to be at high risk of developing dermatitis? .

Dry cleaners

Dye workers

Lathe operators

Hospital workers

Agricultural workers

Dry cleaners Dye workers

The nursing team is providing care for a client. The team leader develops client care plans and coordinates care among the team members. Which member of the team acts as a team leader?

Charge nurse

Registered nurse

Licensed practical nurse

Unlicensed assistive personnel

Registered nurse

Which action by the nurse is appropriate when caring for an elderly client admitted to a healthcare facility?

Ensure the room is brightly lit.

Speak to the client in a loud voice.

Stand close to the client’s ear while speaking.

Invite a family member to join the conversation.

Invite a family member to join the conversation.

According to Kübler-Ross, during which stage of grieving are individuals with serious health problems most likely to seek other medical opinions?

Anger

Denial

Bargaining

Depression

Denial

While reviewing a client’s prescriptions, the nurse finds that one of the prescribed drugs is redundant and notifies the primary healthcare provider. Which attitude of critical thinking does the nurse exhibit?

Curiosity

Risk taking

Thinking independently

Responsibility and authority

Risk taking

A client is being treated for influenza A (H1N1). The nurse has provided instructions to the client about how to decrease the risk of transmission to others. Which patient statement indicates a need for further instruction?

"I should practice respiratory hygiene/cough etiquette."

"I should avoid contact with the elderly or children."

"I should obtain a pneumococcal vaccination each year."

"I should allow visitors for short periods of time only."

"I should obtain a pneumococcal vaccination each year."

The nurse is assessing a young couple planning to start a family. What should the nurse tell the couple about the change that they will have to encounter in their family life-cycle?

Develop intimate peer relationships

Maintain own functions and interests

Realign relationships with extended family

Refocus on midlife material and career issues

Realign relationships with extended family

The registered nurse is teaching a nursing student to use self-disclosure appropriately to promote a therapeutic alliance with the client. Which carative factor is involved in this situation?

Instilling faith-hope

Forming a human-altruistic value system

Promoting and expressing positive and negative feelings

Developing a helping, trusting, and human caring relationship

Forming a human-altruistic value system

The nurse is developing a plan of care for the client who has activity intolerance. In determining the desired client outcomes, what should the nurse do?

Prioritize psychosocial needs over physical needs.

Use the Nursing Outcomes Classification (NOC) only.

Use nursing knowledge to plan outcomes and disregard client and family desires.

Set priorities and outcomes using the client’s and family input.

Set priorities and outcomes using the client’s and family input.

While assessing the motor development of an infant, a nurse asks questions to the mother about the infant’s motor skills. Which statement made by the mother indicates that the infant has developed fine-motor skills?

"My child can stand alone."

"My child walks holding onto furniture."

"My child can place objects into containers."

"My child can sit down from a standing position."

"My child can place objects into containers."

Which risks would the nurse state may be associated with adults who work on or around automobiles? .

Infertility

Asbestosis

Dermatitis

Skin cancer

Nasopharyngeal cancer

Asbestosis Dermatitis

A nurse is assessing a child who is accompanied by a parent. The parent has remarried and has another child from the second marriage. What kind of a family does this child belong to?

Blended family

Extended family

Alternative family

Single-parent family

Blended family

Which nursing intervention can be classified under complex physiologic domain according to the Nursing Interventions Classification (NIC) taxonomy? .

Interventions to restore tissue integrity

Interventions to optimize neurologic functions

Interventions to manage restricted body movements

Interventions to promote comfort using psychosocial techniques

Interventions to provide care before, during, and immediately after surgery

Interventions to restore tissue integrity Interventions to optimize neurologic functions Interventions to provide care before, during, and immediately after surgery

A nurse is reviewing a client’s plan of care. What is the determining factor in the revision of the plan?

Time available for care

Validity of the problem

Method for providing care

Effectiveness of the interventions

Effectiveness of the interventions

Which suggestion should the nurse offer to parents who are concerned about caring for their toddler?

Refrain from giving more than five cups of milk a day.

Allow the toddler to choose a time to take medicine.

Let the toddler watch television if the parent is busy.

Encourage the toddler to drink from two-handled cups.

Encourage the toddler to drink from two-handled cups.

The nurse creates a plan of care for a client with a risk of infection. Which is the most desirable expected outcome for the client?

All nursing functions will be completed by discharge.

All invasive intravenous lines will remain patent.

The client will remain awake, alert, and oriented at all times.

The client will be free of signs and symptoms of infection by discharge.

The client will be free of signs and symptoms of infection by discharge.

A client expresses concern about being exposed to radiation therapy because it can cause cancer. What should the nurse emphasize when informing the client about exposure to radiation?

The dosage is kept at a minimum.

Only a small part of the body is irradiated.

The client’s physical condition is not a risk factor.

Nutritional environment of the affected cells is a risk factor.

Only a small part of the body is irradiated.

What is the most important nursing action involved in caring for a client using medications to manage disease?

Administering the medications

Teaching about the medications

Ensuring adherence to the medication regimen

Evaluating the client’s ability to self-administer medications

Administering the medications

The nurse reviews a medical record and is concerned that the client may develop hyperkalemia. Which disease increases the risk of hyperkalemia?

Crohn disease

Cushing disease

End-stage renal disease

Gastroesophageal reflux disease

End-stage renal disease

The nurse is interviewing a client admitted for uncontrolled diabetes after binging on alcohol for the past 2 weeks. The client states, "I am worried about how I am going to pay my bills for my family while I am hospitalized." Which statement by the nurse would best elicit information from the client?

"You are worried about paying your bills?"

"Don’t worry; your bills will get paid eventually."

"When was the last time you were admitted for hyperglycemia?"

"You really shouldn’t be drinking alcohol because of your diagnosis of diabetes."

"You are worried about paying your bills?"

A nurse considers that communication links people with their surroundings. What should the nurse identify as the most important communication link?

Social

Physical

Materialistic

Environmental

Social

A client calls out to all nursing staff members who pass by the door and asks them to do or get something. How can the nurse best manage this problem while meeting this client’s needs?

Assign one staff member to approach the client regularly and interact with the client.

Close the door to the room so that the client cannot see the staff members as they pass by.

Inform the client that one staff member will come in frequently and check whether the client has any requests.

Arrange for a variety of staff members to take turns going into the room to see whether the client has any requests.

Assign one staff member to approach the client regularly and interact with the client.

A postoperative client says to the nurse, "My neighbor, I mean the person in the next room, sings all night and keeps me awake." The neighboring client has dementia and is awaiting transfer to a nursing home. How can the nurse best handle this situation?

Tell the neighboring client to stop singing.

Close the doors to both clients’ rooms at night.

Give the complaining client the prescribed as-needed sedative.

Move the neighboring client to a room at the end of the hall.

Move the neighboring client to a room at the end of the hall.

The spouse of a comatose client who has severe internal bleeding refuses to allow transfusions of whole blood because they are Jehovah’s Witnesses. The client does not have a Durable Power of Attorney for Healthcare. What action should the nurse take?

Institute the prescribed blood transfusion because the client’s survival depends on volume replacement.

Clarify the reason why the transfusion is necessary and explain the implications if there is no transfusion.

Phone the primary healthcare provider for an administrative prescription to give the transfusion under these circumstances.

Give the spouse a treatment refusal form to sign and notify the primary healthcare provider that a court order now can be sought.

Give the spouse a treatment refusal form to sign and notify the primary healthcare provider that a court order now can be sought.

The chief operational officer (COO) interviews a nurse and asks, "Tell me about your practical experiences in clinical decision making." Which example would the nurse give?

"I palpated the right hip of the client, which appeared red, and noted the warm feeling."

"I identified impaired skin integrity in a pressure ulcer form upon finding redness in the client’s hip."

"I quickly offered a salt recipe to a client with a history hypotension history who suffered from light-headedness and dizziness."

"I assessed weakness and hunger in a client with a history of diabetes history who suffers with light-headedness and blurred vision."

"I identified impaired skin integrity in a pressure ulcer form upon finding redness in the client’s hip."

A nurse in a long-term health care setting will introduce a client who has a PhD to the other clients. The client tells the nurse, "I wish to be called Doctor." How should the nurse respond?

"Your wish will be respected."

"Why do you want to be called Doctor?"

"Residents here call one another by their first names."

"Wouldn’t it be better if the others do not know you are a doctor?"

"Your wish will be respected."

A 93-year-old client in a nursing home has been eating less food during mealtimes. What is the priority nursing intervention?

Substitute a supplemental drink for the meal.

Spoon-feed the client until the food is completely eaten.

Allow the client a longer period of time to complete the meal.

Arrange a consultation for the placement of a gastrostomy tube.

Allow the client a longer period of time to complete the meal.

A client has Clostridium difficile. The nurse is providing discharge instructions related to decreasing the risk of transmission to family members. What would be appropriate to include in the client’s teaching?

Increase fluids.

Increase fiber in the diet.

Wash hands with soap and water.

Wash hands with an alcohol-based hand sanitizer.

Wash hands with soap and water.

A daughter of a Chinese-speaking client approaches a nurse and asks multiple questions while maintaining direct eye contact. What culturally related concept does the daughter’s behavior reflect?

Prejudice

Stereotyping

Assimilation

Ethnocentrism

Assimilation

The nurse at the well baby clinic is assessing the gross motor skills of a five-month-old infant. Which finding is a cause for concern?

The baby has a head lag when pulled to sit.

The baby can turn from the side to the back.

The baby can turn from the abdomen to the back.

The baby supports much of his own weight when he or she is pulled to stand.

The baby has a head lag when pulled to sit.

A client injured in a motor vehicle accident was brought to the emergency and taken immediately for a scan. The client’s family arrives later and asks about the client’s health. What should the nurse tell the client’s family?

"Please do not worry, everything will be alright."

"I am sorry; I do not have any information about the client."

"You will have to wait for the primary healthcare provider."

"Please wait; I will update you as soon as I have any information."

"Please wait; I will update you as soon as I have any information."

What are the benefits of using standard formal nursing diagnostic statements?

Fosters development of nursing knowledge

Allows nurses to communicate with the client

Provides precise definition of the client’s problem

Distinguishes the nurse’s role from that of other care providers

Enables the primary healthcare provider to deliver effective health care

Fosters development of nursing knowledge Provides precise definition of the client’s problem Distinguishes the nurse’s role from that of other care providers

The nurse should place the client in which position to obtain the most accurate reading of jugular vein distention?

Upright at 90 degrees

Supine position

Raised to 30 degrees

Raised to 10 degrees

Raised to 30 degrees

Which intellectual factor would the nurse find appropriate as a dimension for gathering data for a client’s health history?

Attention span

Primary language

Coping mechanisms

Activity and coordination

Attention span

The mother of an 11-month-old infant reports that the baby has allergies. After an assessment, the primary healthcare provider also suspects anemia. Which questions would the primary healthcare provider most likely ask the mother?

Do you use 2% cow’s milk?

Do you breastfeed?

Do you use whole cow’s milk?

Do you use alternate milk products?

Do you provide 18 to 21 ounces of breast milk per day?

Do you use 2% cow’s milk? Do you use whole cow’s milk? Do you use alternate milk products?

The registered nurse (RN) asks the nursing student about theories related to the aging process. Which statement made by the nursing student indicates the theory of continuity?

"The aging process in individuals is genetically programmed."

"Aging individuals withdraw from customary roles."

"The accumulated damage of the aging process leads to physical changes in individuals."

"The personality of an individual remains stable and behavior becomes predictable as they age."

"The personality of an individual remains stable and behavior becomes predictable as they age."

A nurse is teaching a community group about the basics of nutrition. A participant questions why fluoride is added to drinking water. The nurse should respond that it is a necessary element added to drinking water to promote what?

Dental health

Growth and development

Improved hearing

Night vision

Dental health

An 85-year-old client is alert and able to participate in care. The nurse understands that, according to Erikson, a person’s adjustment to the period of senescence will depend largely on adjustment to which developmental stage?

Industry versus inferiority

Identity versus role confusion

Generativity versus stagnation

Autonomy versus shame/doubt

Generativity versus stagnation

Which intervention by the nurse helps the family feel in control when the client is to be discharged home?

Instruct the family to ensure the client’s room is safe.

Ask the family to ensure that the client has only low-fat meals.

Ask the family to coordinate with the staff at the rehabilitation center.

Ensure a family member is confident about changing dressings correctly.

Ensure a family member is confident about changing dressings correctly.

A nurse is reviewing how a hyperglycemic client’s blood glucose can be lowered. The nurse recalls that the chemical that buffers the client’s excessive acetoacetic acid is what?

Potassium

Sodium bicarbonate

Carbon dioxide

Sodium chloride

Sodium bicarbonate

Which standards would the nurse explain are important for critical thinking?

Specific

Fairness

Relevant

Confidence

Independence

Specific Relevant

In an interview, the manager is looking for a nurse who excels in analyticity. Which statement made by the nurse would help him or her to get selected for the new project?

"I am able to make my own decisions."

"I am highly organized and focused in my work."

"I am good at using evidence-based knowledge."

"I am very passionate about acquiring knowledge."

"I am good at using evidence-based knowledge."

Which is an indirect nursing care intervention?

Administering medications

Managing the client’s environment

Counseling the family during a time of grief

Inserting intravenous infusion

Managing the client’s environment

Nursing actions for an older adult should include health education and promotion of self-care. Which is most important when working with an older adult client?

Encouraging frequent naps

Strengthening the concept of ageism

Reinforcing the client’s strengths and promoting reminiscing

Teaching the client to increase calories and focusing on a high-carbohydrate diet

Reinforcing the client’s strengths and promoting reminiscing

An 80-year-old client is admitted to the hospital because of complications associated with severe dehydration. The client’s daughter asks the nurse how her mother could have become dehydrated, because she is alert and able to care for herself. The nurse’s best response is:

"The body’s fluid needs decrease with age because of tissue changes."

"Access to fluid may be insufficient to meet the daily needs of the older adult."

"Memory declines with age, and the older adult may forget to ingest adequate amounts of fluid."

"The thirst reflex diminishes with age, and therefore the recognition of the need for fluid is decreased."

"The thirst reflex diminishes with age, and therefore the recognition of the need for fluid is decreased."

To decrease abdominal distention following a client’s surgery, what actions should the nurse take?

Encourage ambulation.

Give sips of ginger ale.

Provide a straw for drinking.

Offer an opioid analgesic.

Encourage ambulation.

A nurse applies a cold pack to treat an acute musculoskeletal injury. Cold therapy decreases pain by doing what?

Promoting analgesia and circulation

Numbing the nerves and dilating the blood vessels

Promoting circulation and reducing muscle spasms

Causing local vasoconstriction, preventing edema and muscle spasms

Causing local vasoconstriction, preventing edema and muscle spasms

Which description is most appropriate for the family centered care approach?

The nursing care is focused on the client as an individual.

A collaborative plan of care is developed to achieve optimal health.

The healthcare provider is the expert in developing a plan of care.

The nursing care is based solely on standards of practice.

A collaborative plan of care is developed to achieve optimal health.

A registered nurse is teaching a nursing student about how to safely use a urinary catheter. Which statement made by the nursing student indicates ineffective learning?

"I will avoid the pooling of urine in the tubing."

"I will avoid prolonged clamping of the tubing."

"I will avoid draining urine from the tubing before ambulation."

"I will avoid raising the drainage tube above the level of the bladder."

"I will avoid draining urine from the tubing before ambulation."

A client is receiving therapy that includes a radioactive sealed implant. What nursing intervention should be implemented to protect against exposure to radiation?

Wearing a dosimeter film badge at all times

Limiting exposure to the client to 1 hour daily

Using long-handled forceps to retrieve a dislodged implant

Ensuring that visitors maintain a minimum distance of 3 feet from the client

Using long-handled forceps to retrieve a dislodged implant

Which right of delegation refers to the giving of clear, concise descriptions of a task to the delegatee?

Right task

Right person

Right supervision

Right communication

Right communication

The registered nurse is teaching the student nurse about writing nursing interventions. Which intervention written by the student nurse indicates effective learning?

"Turn the client every 2 hours."

"Perform blood glucose measurements regularly."

"Change the client’s dressing once a shift: 6 AM—2 PM—10 PM."

"Irrigate the wound with 100 mL normal saline until clear: 6 AM—2 PM—8 PM."

"Irrigate the wound with 100 mL normal saline until clear: 6 AM—2 PM—8 PM."

A nurse teaches a client about wearing thigh-high antiembolism elastic stockings. What would be appropriate to include in the instructions?

"You do not need to wear them while you are awake, but it is important to wear them at night."

"You will need to apply them in the morning before you lower your legs from the bed to the floor."

"If they bother you, you can roll them down to your knees while you are resting or sitting down."

"You can apply them either in the morning or at bedtime, but only after the legs are lowered to the floor."

"You will need to apply them in the morning before you lower your legs from the bed to the floor."

Alternative therapy measures have become increasingly accepted within the past decade, especially in the relief of pain. Which methods qualify as alternative therapies for pain?

Prayer

Hypnosis

Medication

Aromatherapy

Guided imagery

Prayer Hypnosis Aromatherapy Guided imagery

Which action of the nurse would be most important to convey interest in starting a conversation with a client who has hearing loss?

Smiling while seeing the client

Nodding head in front of the client

Making eye contact with the client

Leaning forward towards the client

Making eye contact with the client

A high-protein diet is recommended for a client recovering from a fracture. The nurse recalls that the rationale for a high-protein diet is to do what?

Promote gluconeogenesis.

Produce an antiinflammatory effect.

Promote cell growth and bone union.

Decrease pain medication requirements.

Promote cell growth and bone union.

Which application in practice would best suit the critical thinking attitude called integrity?

To look for different approaches

To follow the standards of practice in care

To always be well prepared before performing a nursing activity

To be able to recognize personal opinions that may conflict with a client

To be able to recognize personal opinions that may conflict with a client

The registered nurse is teaching a nursing student about how to educate a client who is having trouble adapting to his or her illness. Which statement made by the nursing student indicates the need for further education?

"I should motivate the client’s interest in learning."

"I should assume that the family is involved."

"I should identify the client’s willingness to learn."

"I should give information about how to perform skills within the home."

"I should assume that the family is involved."

What interventions should the nurse follow when giving health education to an elderly client?

Assess the client for pain before teaching.

Take down notes while talking to the client.

Ensure the client is not preoccupied or anxious.

Teach one concept at a time according to the client’s interest.

Teach a family caregiver if the client does not respond quickly.

Assess the client for pain before teaching. Ensure the client is not preoccupied or anxious. Teach one concept at a time according to the client’s interest.

A client requests information about the prescribed medication regimen. What is the best response by the nurse?

Give a computer printout about the medication to the client.

Ask the client to state what is already known about the medication.

Advise talking to the primary healthcare provider to seek information about the medication.

Delegate the task of sharing information about the medication to the licensed practical nurse.

Ask the client to state what is already known about the medication.

Which intervention does the nurse implement to develop a caring relationship with the client’s family?

Deciding healthcare options for the client

Identifying the client’s family members and their roles

Declining to inform the client’s family after performing a procedure

Refraining from discussing the client’s health with the family

Identifying the client’s family members and their roles

A nurse is caring for a client diagnosed with methicillin-resistant Staphylococcus aureus (MRSA) in the urine. The healthcare provider orders an indwelling urinary catheter to be inserted. Which precaution should the nurse take during this procedure?

Droplet precautions

Reverse isolation

Surgical asepsis

Medical asepsis

Surgical asepsis

Which statement defines the term family resiliency?

Family resiliency is the uniqueness of each family.

Family resiliency is the ability of the family to cope with stressors.

Family resiliency is the intrafamilial system of support and structure.

Family resiliency is the ability of the family to transcend.

Family resiliency is the ability of the family to cope with stressors.

A nurse is assessing a middle-aged client whose children have left home in search of work. The client is trying to adjust to these family changes. Which family life-cycle stage is the client going through?

Family in later life

Family with adolescents

Unattached young adult

Launching children and moving on

Launching children and moving on

A client suffering from cancer is at the last stage of life. Which actions should be performed by the nurse to support the client’s family members?

Helping the family to set up home care

Taking time to make sure that the family is comfortable

Staying with the client in the absence of family members

Giving the family about the information of dying process

Making sure that the family knows about what to do at the time of death

Helping the family to set up home care Giving the family about the information of dying process Making sure that the family knows about what to do at the time of death

The nurse instructs a client that, in addition to building bones and teeth, calcium is also important for what?

Bile production

Blood production

Blood clotting

Digestion of fats

Blood clotting

A nurse on the medical-surgical unit tells other staff members, "That client can just wait for the lorazepam; I get so annoyed when people drink too much." What does this nurse’s comment reflect?

Demonstration of a personal bias

Problem solving based on assessment

Determination of client acuity to set priorities

Consideration of the complexity of client care

Demonstration of a personal bias

What are the advantages of the team nursing model of providing nursing care?

The nursing care conferences help to solve client problems.

The client satisfaction is highest compared to other models.

The model provides a high level of autonomy for the team leader.

The registered nurses are responsible for planning care for each client.

The model facilitates a high level of collaboration between team members.

The nursing care conferences help to solve client problems. The model provides a high level of autonomy for the team leader. The model facilitates a high level of collaboration between team members.

A client has a platelet count of 49,000/mL (40 × 10 9/L). The nurse should instruct the client to avoid which activity?

Ambulation

Blowing the nose

Visiting with children

The semi-Fowler position

Blowing the nose

A home health nurse on a first visit checks the client’s vital signs and obtains a blood sample for an international normalized ratio (INR). After these tasks are completed, the client asks the nurse to straighten the blankets on the bed. What is the nurse’s most appropriate response?

"I would, but my back hurts today."

"Okay. It will be my good deed for the day."

"Of course. I want to do whatever I can for you."

"I would like to, but it is not in my job description."

"Of course. I want to do whatever I can for you."

When suctioning a client with a tracheostomy, an important safety measure for the nurse is to do what?

Hyperventilate the client with room air before suctioning.

Apply suction only as the catheter is being withdrawn.

Insert the catheter until the cough reflex is stimulated.

Remove the inner cannula before inserting the suction catheter.

Apply suction only as the catheter is being withdrawn.

A client has undergone a subtotal thyroidectomy. The client is being transferred from the postanesthesia care unit/recovery area to the inpatient nursing unit. What emergency equipment is most important for the nurse to have available for this client?

A defibrillator

An IV infusion pump

A tracheostomy tray

An electrocardiogram (ECG) monitor

A tracheostomy tray

On the second day of hospitalization a client is discussing with the nurse concerns about unhealthy family relationships. During the nurse-client interaction the client begins to talk about a job problem. The nurse’s response is, "Let’s go back to what we were just talking about." What therapeutic communication technique did the nurse use?

Focusing

Restating

Exploring

Accepting

Focusing

The nurse is preparing discharge instructions for a client who has begun to demonstrate signs of early Alzheimer dementia. The client lives alone. The client’s adult children live nearby. According to the prescribed medication regimen, the client is to take medications six times throughout the day. What is the priority nursing intervention to assist the client with taking the medication?

Contact the client’s children and ask them to hire a private duty aide who will provide round-the-clock care.

Develop a chart for the client, listing the times the medication should be taken.

Contact the primary healthcare provider and discuss the possibility of simplifying the medication regimen.

Instruct the client and client’s children to put medications in a weekly pill organizer.

Contact the primary healthcare provider and discuss the possibility of simplifying the medication regimen.

The nurse assessing an adult understands that the client is experiencing a midlife crisis. Which factor should the nurse attribute to this condition?

The client is seeking an occupational direction.

The client is examining life goals and relationships.

The client is directing energy towards achievements.

The client is sharing responsibilities in a two-career family.

The client is examining life goals and relationships.

Which example best demonstrates humility in a critical thinker?

A nurse questions the occurrence of pneumonia in a client who has a history of smoking.

A nurse finds a client in pain and asks specific and detailed questions about the pain in order to provide pain relief.

A nurse in a surgical scrub touches a contaminated surface and performs the whole process of surgical scrub again.

A nurse accepts his or her lack of knowledge regarding stem cell transplantation and seeks opportunities for learning.

A nurse accepts his or her lack of knowledge regarding stem cell transplantation and seeks opportunities for learning.

The nurse receives a report on a newly admitted client who is positive for Clostridium difficile. Which category of isolation would the nurse implement for this client?

Airborne precautions

Droplet precautions

Contact precautions

Protective environment

Contact precautions

A nurse is providing morning hygiene to a bedridden client who was admitted for exacerbation of chronic obstructive pulmonary disease (COPD). What is the priority nursing intervention when the client becomes short of breath during the care?

Obtain a pulse oximeter to determine the client’s oxygen saturation level.

Put the client in a high Fowler position.

Darken the lights and provide a rest period of at least 15 minutes.

Continue the hygiene activities while reassuring the client.

Put the client in a high Fowler position.

A client is being admitted to a medical unit with a diagnosis of pulmonary tuberculosis. The nurse should assign the client to which type of room?

Private room

Semiprivate room

Room with windows that can be opened

Negative-airflow room

Negative-airflow room

A client is being discharged from the hospital with an indwelling urinary catheter. The client asks about the best way to prevent infection and keep the catheter clean. Which would be appropriate for the nurse to include in the client teaching?

Once a day, clean the tubing with a mild soap and water, starting at the drainage bag and moving toward the insertion site.

After cleaning the catheter site, it is important to keep the foreskin pushed back for 30 minutes to ensure adequate drying.

Clean the insertion site daily using a solution of one part vinegar to two parts water.

Replace the drainage bag with a new bag once a week.

Replace the drainage bag with a new bag once a week.

Which intellectual factor would the nurse find appropriate as a dimension for gathering data for a client’s health history?

Attention span

Primary language

Coping mechanisms

Activity and coordination

Attention span

What should the nurse educator instruct a graduate nurse who is seeking employment?

Be a role model to the nursing staff

Attend workshops and conferences

Motivate other educators on the nursing unit

Work on developing effective communication

Approach institutions that provide mentoring

Attend workshops and conferences Work on developing effective communication Approach institutions that provide mentoring

A client’s arterial blood gas report indicates the pH is 7.52, PCO 2 is 32 mm Hg, and HCO 3 is 24 mEq/L. What does the nurse identify as a possible cause of these results?

Airway obstruction

Inadequate nutrition

Prolonged gastric suction

Excessive mechanical ventilation

Excessive mechanical ventilation

A client is to receive a transfusion of packed red blood cells (PRBCs). The nurse should prepare for the transfusion by priming the blood IV tubing with which solution?

Lactated Ringer solution

5% dextrose and water

0.9% normal saline

0.45% normal saline

0.9% normal saline

After several weeks of caring for clients who are in the terminal stage of illness, the nurse becomes aware of feeling depressed when coming to work. What should the nurse do?

Talk with other nurses on the unit.

Take several personal days off from work.

Limit emotional involvement with the clients.

Request a transfer to another area of the hospital.

Talk with other nurses on the unit.

A client has a paracentesis, and the healthcare provider removes 1500 mL of fluid. To monitor for a serious postprocedure complication, what should the nurse assess for?

Dry mouth

Tachycardia

Hypertensive crisis

Increased abdominal distention

Tachycardia

The healthcare provider prescribes a low-fat, 2-gram sodium diet for a client with hypertension. The nurse should explain that the purpose of restricting sodium is to do what?

Chemically stimulate the loop of Henle

Diminish the thirst response of the client

Prevent reabsorption of water in the distal tubules

Cause fluid to move toward the interstitial compartment

Prevent reabsorption of water in the distal tubules

The registered nurse is teaching a nursing student about nursing care principles for cognitively impaired older adults. Which statement made by the nursing student indicates a need for further education?

"I should encourage fluid intake."

"I should provide conditional positive support."

"I should promote social interaction based on abilities."

"I should provide ongoing assistance to family caregiver."

"I should provide conditional positive support."

A registered nurse is caring for a client who is on isolation precautions. Which tasks can be safely assigned to the nursing assistive personnel? .

Assessing vital signs

Administering injections

Assessing wound drainage

Bringing equipment to the client’s room

Transporting the client to a diagnostic test

Bringing equipment to the client’s room Transporting the client to a diagnostic test

Which infant is likely to need iron supplementation throughout the first year?

A full-term infant who is breast feeding.

A full-term infant who is receiving formula.

A breastfed infant who is four months old.

A breastfed infant who is six months old.

A full-term infant who is receiving formula.

A nurse is caring for a client for whom segmental postural drainage treatments are prescribed. The nurse should avoid scheduling the treatment at what time?

At bedtime

After a meal

One hour before a meal

One hour after awakening

After a meal

A nurse is evaluating the effectiveness of treatment for a client with excessive fluid volume. What clinical finding indicates that treatment has been successful?

Clear breath sounds

Positive pedal pulses

Normal potassium level

Decreased urine specific gravity

Clear breath sounds

Which actions by the nurse help set the stage for a patient-centered interview during the first visit after admission to the healthcare facility?

Close the door after entering the room.

Greet the client using his or her last name.

Open the curtains to allow plenty of light in the room.

Introduce oneself with a smile and explain the reason for the visit.

Obtain an authorization from the client after the interview.

Close the door after entering the room. Greet the client using his or her last name. Introduce oneself with a smile and explain the reason for the visit.

The nurse is reviewing the data of clients with pre-hypertension. Which client is at risk of stage 1 hypertension based on the given data?

Client B

The heartbeat assessment of four clients is given below.
Which client is at an increased risk for right-sided heart failure?

Client A

The nurse performs a respiratory assessment and auscultates breath sounds that are high pitched, creaking, and accentuated on expiration. Which term best describes the findings?

Rhonchi

Wheezes

Pleural friction rub

Bronchovesicular

Wheezes

A client suspected to have a prostate disorder is encouraged to have a rectal examination. What position of the client will facilitate a rectal examination by the registered nurse (RN)?

Sims position

Prone position

Dorsal recumbent position

Lateral recumbent position

Sims position

While inspecting the external eye structure of a client, a nurse finds bulging of the eyes. Which condition can be suspected in the client?

Eye tumors

Hypothyroidism

Hyperthyroidism

Neuromuscular injury

Hyperthyroidism

A client with a history of cardiac dysrhythmias is admitted to the hospital due to a fluid volume deficit caused by a pulmonary infection. The registered nurse is assessing the vital signs recorded by the student nurse. Which vital sign assessments require reassessment based on the data given by the student nurse?

Respiratory rate of 14 breaths/minute

Blood pressure of 120/80 mmHg

Oxygen saturation of 95%

Temporal temperature of 37.4 °C

Radial pulse rate of 72 and irregular

Respiratory rate of 14 breaths/minute Blood pressure of 120/80 mmHg Oxygen saturation of 95%

While caring for a female client, the nursing student feels tenderness and a lump in the client’s breast. The nursing student tells the registered nurse, "I think this client has breast cancer." Which statements of the registered nurse would be appropriate in accordance with the knowing element of Swanson’s theory?

"Try to comfort the client."

"Avoid making assumptions."

"Assess the client thoroughly."

"Check for other signs of breast cancer."

"Try to provide support and care to the client."

Avoid making assumptions." "Assess the client thoroughly." "Check for other signs of breast cancer."

A client is hospitalized for treatment of severe hypertension. Captopril and alprazolam are prescribed. Shortly after admission, the client says, "I don’t think any of you know what you are doing. You are just guessing what I need." What does the nurse determine as the probable cause of this behavior?

Denial of illness

Fear of the health problem

Response to cerebral anoxia

Reaction to the antihypertensive drug

Fear of the health problem

What should a nurse recommend to best help a client during the period immediately after a spouse’s death?

Crisis counseling

Family counseling

Marital counseling

Bereavement counseling

Bereavement counseling

An adolescent is taken to the emergency department of the local hospital after stepping on a nail. The puncture wound is cleansed and a sterile dressing applied. The nurse asks about tetanus immunization. The adolescent responds that all immunizations are up to date. Penicillin is administered, and the client is sent home with instructions to return if there is any change in the wound area. A few days later, the client is admitted to the hospital with a diagnosis of tetanus. Legally, what is the nurse’s responsibility in this situation?

The nurse’s judgment was adequate, and the client was treated accordingly.

The possibility of tetanus was not foreseen because the client was immunized.

Nurses should routinely administer immunization against tetanus after such an injury.

Assessment by the nurse was incomplete, and as a result the treatment was insufficient.

Assessment by the nurse was incomplete, and as a result the treatment was insufficient.

While assessing the nails of a client with diabetes, the nurse finds that the skin on the client’s hands and feet are dry due to infection. What could be the reason for this dryness?

Applying moisturizing lotion between toes

Cutting nails after soaking them for 10 minutes in warm water

Cutting nails straight across and even with the tops of the fingers or toes

Using sharp objects to poke or dig under the toenail or around the cuticle

Cutting nails after soaking them for 10 minutes in warm water **Normally, nails should be cut after soaking them in warm water for 10 minutes. This action should not be performed for diabetic patients because soaking the nails will dry out the hands and feet, which may lead to infection. Applying moisturizing lotion between the toes will promote microorganism growth; it will not dry the skin. Cutting nails straight across and even with the tops of the fingers or toes is the proper way to maintain nail hygiene. Diabetic clients are advised not to use sharp objects to poke or dig under the toenails or around the cuticles to avoid injury to the skin.

The nurse assessing an adult understands that the client is experiencing a midlife crisis. Which factor should the nurse attribute to this condition?

The client is seeking an occupational direction.

The client is examining life goals and relationships.

The client is directing energy towards achievements.

The client is sharing responsibilities in a two-career family.

The client is examining life goals and relationships.

At the beginning of the shift at 7:00 am, a client has 650 mL of normal saline solution left in the intravenous bag, which is infusing at 125 mL/hr. At 9:30 am the healthcare provider changes the IV solution to lactated Ringer solution, which is to infuse at 100 mL/hr. What total amount of intravenous solution should the client have received by the end of the 8-hour shift? Record your answer using a whole number.

863 mL

What type of functional health pattern would the nurse explain describes values and goals?

Value-belief pattern

Role-relationship pattern

Self-perception-self-concept pattern

Health perception-health management pattern

Value-belief pattern

When monitoring a client 24 to 48 hours after abdominal surgery, the nurse should assess for which problem associated with anesthetic agents?

Colitis

Stomatitis

Paralytic ileus

Gastrocolic reflux

Paralytic ileus

A client requests information about the prescribed medication regimen. What is the best response by the nurse?

Give a computer printout about the medication to the client.

Ask the client to state what is already known about the medication.

Advise talking to the primary healthcare provider to seek information about the medication.

Delegate the task of sharing information about the medication to the licensed practical nurse.

Ask the client to state what is already known about the medication.

Which approach is a comforting approach that communicates concern and support?

Touch

Listening

Knowing the client

Providing a positive presence

Touch

When planning discharge teaching for a young adult, the nurse should include the potential health problems common in this age group. What should the nurse include in this teaching plan?

Kidney dysfunction

Cardiovascular diseases

Eye problems, such as glaucoma

Accidents, including their prevention

Accidents, including their prevention

A nurse is taking care of a client who has chronic back pain as a result of a work injury. What nursing considerations should be made when determining the client’s plan of care?

Ask the client about the acceptable level of pain.

Eliminate all activities that precipitate the pain.

Administer the pain medications regularly around the clock.

Use a different pain scale each time to promote patient education.

Assess the client’s pain every 15 minutes.

Ask the client about the acceptable level of pain. Administer the pain medications regularly around the clock.

A client with dementia who feels highly anxious and confused believes that the current day is actually different than what it is. Which statement made by the nurse is an example of validation therapy?

"No, try to be in your sense of reality."

"Yes, today is the day that you just mentioned."

"You should try improving your awareness level."

"Try to recall your past memories associated with the day."

"Yes, today is the day that you just mentioned."

A mother is worried about the sudden behavioral changes in her child. The child has suddenly developed a fear of certain people and places. The child’s school performance is declining rapidly, and the child has developed poor relationships with his or her peers. After assessing the physical findings of the child, the nurse suspects child abuse. Which physical findings might have led the nurse to this suspicion?

Sunken eyes and loss of weight

Uncommunicative and uninteractive with others

Foreign bodies in the rectum, urethra, or vagina

Strangulation marks on neck from rope burns or bruises

Foreign bodies in the rectum, urethra, or vagina

The nurse manager asks the nurse, "How would you implement clinical decision making in a group of clients?" Which answer provided by the nurse shows effective critical thinking?

"I will avoid involving clients as decision-makers and participants in care."

"I will discuss complex cases with other members of the healthcare team."

"I will identify the nursing diagnoses and collaborative problems of each client."

"I will consider the period it takes to care for clients whose problems have higher priority."

"I will decide to perform activities individually to resolve more than one client problem at a time."

"I will discuss complex cases with other members of the healthcare team." "I will identify the nursing diagnoses and collaborative problems of each client." "I will consider the period it takes to care for clients whose problems have higher priority."

When caring for a client with varicella and disseminated herpes zoster, the nurse should implement which types of precautions?

Airborne

Contact

Droplet

Hazardous wastes

Standard

Airborne Contact Standard

The nurse should suspect that a client who had a recent myocardial infarction is experiencing denial when the client does what?

Attempts to minimize the illness

Lacks an emotional response to the illness

Refuses to discuss the condition with the client’s spouse

Expresses displeasure with the prescribed activity program

Attempts to minimize the illness

The nurse recognizes that a common conflict experienced by older adults is the conflict between what?

Youth and old age

Retirement and work

Independence and dependence

Wishing to die and wishing to live

Independence and dependence

A dying client is coping with feelings regarding impending death. The nurse bases care on the theory of death and dying by Kübler-Ross. During which stage of grieving should the nurse primarily use nonverbal interventions?

Anger

Denial

Bargaining

Acceptance

Acceptance

A 50-year-old client with a 30-year history of smoking reports a chronic cough and shortness of breath related to chronic obstructive pulmonary disease (COPD). The clinical data on admission are as follows: a heart rate of 100, a blood pressure of 138/82, a respiratory rate of 32, a tympanic temperature 36.8 °C, and an oxygen saturation of 80%. Which vital signs obtained by the nurse during the therapy indicates a positive outcome?

Radial pulse: 70

Temperature: 37 °C

Respiratory rate: 14

Blood pressure: 110/70

Oxygen saturation: 92%

Respiratory rate: 14 Blood pressure: 110/70 Oxygen saturation: 92%

A client undergoes a bowel resection. When assessing the client 4 hours postoperatively, the nurse identifies which finding as an early sign of shock?

Respirations of 10

Urine output of 30 mL/hour

Lethargy

Restlessness

Restlessness

The registered nurse is teaching a nursing student about providing care to an older adult with dementia. Which statement made by the nursing student indicates a need for further education?

"I should serve food that is easy to eat."

"I should assist the client with eating."

"I should monitor weight and food intake once in a month."

"I should offer food supplements that are tasty and easy to swallow."

"I should monitor weight and food intake once in a month."

The registered nurse is teaching the student nurse about writing nursing interventions. Which intervention written by the student nurse indicates effective learning?

"Turn the client every 2 hours."

"Perform blood glucose measurements regularly."

"Change the client’s dressing once a shift: 6 AM—2 PM—10 PM."

"Irrigate the wound with 100 mL normal saline until clear: 6 AM—2 PM—8 PM."

"Irrigate the wound with 100 mL normal saline until clear: 6 AM—2 PM—8 PM."

Which sites would be safe and inexpensive for temperature measurement?

Skin

Oral

Axilla

Rectal

Tympanic membrane

Skin Axilla

A day after an explanation of the effects of surgery to create an ileostomy, a 68-year-old client remarks to the nurse, "It will be difficult for my wife to care for a helpless old man." This comment by the client regarding himself is an example of Erikson’s conflict of what?

Initiative versus guilt

Integrity versus despair

Industry versus inferiority

Generativity versus stagnation

Integrity versus despair

While a nurse is providing food to a client in traction, the client reports feeling uncomfortable from being in the same position. Which nursing intervention is priority in this situation?

Repositioning the client

Offering basic hygiene measures

Assisting the client with the meal

Providing health teaching to the client

Repositioning the client

According to Kübler-Ross, during which stage of grieving are individuals with serious health problems most likely to seek other medical opinions?

Anger

Denial

Bargaining

Depression

Denial

While performing a physical assessment, the nurse notices a minute, nonpalpable change in the skin color of a client. What might be the type of skin lesion involved?

Wheal

Papule

Vesicle

Macule

A macule is a flat, nonpalpable change in skin color, which is smaller than 1 cm.

A client who is dying jokes about the situation even though the client is becoming sicker and weaker. Which is the most therapeutic response by the nurse?

"Why are you always laughing?"

"Your laughter is a cover for your fear."

"Does it help to joke about your illness?"

"The person who laughs on the outside cries on the inside."

"Does it help to joke about your illness?"

The client reports difficulty in breathing. The nurse auscultates lung sounds and assesses the respiratory rate. What is the purpose of the nurse’s action?

Data collection

Data validation

Data clustering

Data interpretation

Data collection

The nurse is assisting with the end-of-life care of an older adult. Which activity is performed when the nurse views family as context?

Assess the resources available to the family

Meet the client’s family’s comfort and nutritional needs

Meet the client’s comfort, hygiene and nutritional needs

Determine the family’s need for rest and their stage of coping

When viewing family as context, the nurse mainly focuses on the client’s comfort, hygiene, and nutritional needs. Family as context means focusing on the health and development of a client.

What principle of teaching specific to an older adult should the nurse consider when providing instruction to such a client recently diagnosed with diabetes mellitus?

Knowledge reduces general anxiety.

Capacity to learn decreases with age.

Continued reinforcement is advantageous.

Readiness of the learner precedes instruction.

Continued reinforcement is advantageous.

The nurse administers a pneumococcal vaccine to a 70-year-old client. The client asks "Will I have to get this every year like I do with the flu shot?" How should the nurse respond?

"You need to receive the pneumococcal vaccine every other year."

"The pneumococcal vaccine should be received in early autumn every year."

"You should get the flu and pneumococcal vaccines at your annual physical examination."

"It is unnecessary to have any follow-up injections of the pneumococcal vaccine after this dose."

"It is unnecessary to have any follow-up injections of the pneumococcal vaccine after this dose."

A client experiencing chills and fever is admitted to the hospital. After assessing the client’s vitals and medical history, the nurse concluded that the client’s fever pattern is remittent. Which assessment finding led to this conclusion?

The client’s temperature returns to an acceptable value at least once in the past 24 hours

The client’s fever spikes and falls without a return to normal temperature levels

Periods of febrile episodes and periods with acceptable temperature values occur

The client has a constant body temperature continuously above 38°C with minimal fluctuation

The client’s fever spikes and falls without a return to normal temperature levels

The nurse pulls up on the client’s skin and releases it to determine whether the skin returns immediately to its original position. What is the nurse assessing for?

Pain tolerance

Skin turgor

Ecchymosis formation

Tissue mass

Skin turgor

The nurse recognizes that which are important components of a neurovascular assessment?

Orientation

Capillary refill

Pupillary response

Respiratory rate

Pulse and skin temperature

Movement and sensation

Capillary refill Pulse and skin temperature Movement and sensation

Which assessing technique involves tapping a client’s skin with the fingertips to cause vibrations in the underlying tissues?

Palpation

Inspection

Percussion

Auscultation

Percussion

A nurse is applying a dressing to a client’s surgical wound using sterile technique. While engaging in this activity, the nurse accidentally places a moist sterile gauze pad on the cloth sterile field. What physical principle is applicable for causing the sterile field to become contaminated?

Dialysis

Osmosis

Diffusion

Capillarity

Capillarity

An older adult client who is receiving chemotherapy for cancer has severe nausea and vomiting and becomes dehydrated. The client is admitted to the hospital for rehydration therapy. Which nursing actions have specific gerontologic implications the nurse must consider?

Assessment of skin turgor

Documentation of vital signs

Assessment of intake and output

Administration of antiemetic drugs

Replacement of fluid and electrolytes

Assessment of skin turgor Administration of antiemetic drugs Replacement of fluid and electrolytes

A 50-year-old client being seen for a routine physical asks why a stool specimen for occult blood testing has been prescribed when there is no history of health problems. What is an appropriate nursing response?

"You will need to ask your healthcare provider; it is not part of the usual tests for people your age."

"There must be concern of a family history of colon cancer; that is a primary reason for an occult blood stool test."

"It is performed routinely starting at your age as part of an assessment for colon cancer."

"There must have been a positive finding after a digital rectal examination performed by your healthcare provider."

"It is performed routinely starting at your age as part of an assessment for colon cancer."

Which nursing intervention can be classified under complex physiologic domain according to the Nursing Interventions Classification (NIC) taxonomy? .

Interventions to restore tissue integrity

Interventions to optimize neurologic functions

Interventions to manage restricted body movements

Interventions to promote comfort using psychosocial techniques

Interventions to provide care before, during, and immediately after surgery

Interventions to restore tissue integrity Interventions to optimize neurologic functions Interventions to provide care before, during, and immediately after surgery

During the beginning phase of a therapeutic relationship, why is a clear understanding of participants’ roles important?

The client should understand what will be discussed.

The client will know that the nurse is trying to be helpful.

The client needs to know what to expect from the relationship.

The client will be able to be prepared for termination of the relationship.

The client needs to know what to expect from the relationship.

A nurse is assisting a client to transfer from the bed to a chair. What should the nurse do to widen the client’s base of support during the transfer?

Spread the client’s feet away from each other.

Move the client on the count of three.

Instruct the client to flex the muscles of the internal girdle.

Stand close to the client when assisting with the move.

Spread the client’s feet away from each other.

The nurse is providing postoperative care to a client who had a submucosal resection (SMR) for a deviated septum. The nurse should monitor for what complication associated with this type of surgery?

Occipital headache

Periorbital crepitus

Expectoration of blood

Changes in vocalization

Expectoration of blood

A client who experienced extensive burns is receiving intravenous fluids to replace fluid loss. The nurse should monitor for which initial sign of fluid overload?

Crackles in the lungs

Decreased heart rate

Decreased blood pressure

Cyanosis

Crackles in the lungs

A client develops an allergic reaction when a student nurse is performing a physical assessment. Which statement made by the student nurse in response to this incident indicates the need for further teaching?

"Type I immune response to latex has an immediate onset."

"Type I immune reaction to latex leads to release of IgE antibodies."

"The client’s first exposure to latex will cause a type IV allergic reaction."

"Type IV immune response to latex occurs after 12 to 48 hours after exposure."

"The client’s first exposure to latex will cause a type IV allergic reaction."

A nurse is caring for a client who has been admitted with right-sided heart failure. The nurse notes that the client has dependent edema around the area of the feet and ankles. To characterize the severity of the edema, the nurse presses the medial malleolus area, noting an 8 mm depression after release. How should the edema be documented?

1+

2+

3+

4+

4+

A client is admitted to the hospital for an elective surgical procedure. The client tells a nurse about the emotional stress of recently disclosing being a homosexual to family and friends. What is the nurse’s first consideration when planning care?

Exploring the client’s emotional conflict

Identifying personal feelings toward this client

Planning to discuss this with the client’s family

Developing a rapport with the client’s healthcare provider

Identifying personal feelings toward this client

A registered nurse is teaching a nursing student about the third heart sound (S 3). Which statement given by the nursing student indicates a need for further education?

"S 3 is heard in clients with heart failure."

"S 3 is normal in pregnant women."

"S 3 is abnormal in adults over 31 years of age."

"S 3 is normal in children and young adults."

"S 3 is normal in pregnant women."

A nurse in a long-term health care setting will introduce a client who has a PhD to the other clients. The client tells the nurse, "I wish to be called Doctor." How should the nurse respond?

"Your wish will be respected."

"Why do you want to be called Doctor?"

"Residents here call one another by their first names."

"Wouldn’t it be better if the others do not know you are a doctor?"

"Your wish will be respected."

What critical thinking skill is applicable when knowledge and experience is used to care for clients?

Analysis

Evaluation

Explanation

Interpretation

Explanation

Which assessment should the nurse exclude when dealing with a client with receptive and expressive aphasia?

Ask the client to read simple sentences aloud

Point to a familiar object and ask the client to name it

Test the mental status by asking for feedback from the client

Ask the client to respond to simple verbal commands such as "Stand up"

Test the mental status by asking for feedback from the client **Receptive and expressive aphasia are the two types of aphasia. A client with receptive is unable to understand written or verbal speech. A client with expressive aphasia understands written and verbal speech but cannot write or speak appropriately. A client with aphasia may not have the mental ability to give feedback; asking for feedback is ineffective. Asking the client to read simple sentences aloud is an effective way of dealing with this client. Pointing to a familiar object and asking the client to name it is also effective. A client with aphasia can understand simple verbal commands.

A nurse is providing colostomy care to a client with a nosocomial infection caused by methicillin-resistant Staphylococcus aureus (MRSA). Which personal protective equipment (PPE) should the nurse use?

Gloves

Gown

Mask

Goggles

Shoe covers

Hair bonnet

Gloves Gowns Goggles

A client becomes anxious after being scheduled for a colostomy. What is the most effective way for the nurse to help the client?

Administer the prescribed as needed (PRN) sedative.

Encourage the client to express feelings.

Explain the postprocedure course of treatment.

Reassure the client that there are others with this problem.

Encourage the client to express feelings.

What is the appropriate blood pressure of a 12-year-old client?

95/65 mm Hg

105/65 mm Hg

110/65 mm Hg

119/75 mm Hg

110/65 mm Hg **A 12-year-old client typically has a blood pressure of 110/65 mm Hg. A 1-year-old client would typically have a blood pressure of 95/65 mm Hg. A 6-year-old client would typically have a blood pressure of 105/65 mm Hg. A 14- to 17-year-old client has a typical blood pressure of 119/75 mm Hg.

What principle must a nurse consider when caring for a client with a closed wound drainage system?

Gravity causes fluids to flow down a pressure gradient.

Fluid flow rate is determined by the diameter of the lumen.

Siphoning causes fluids to flow from one level to a lower level.

Fluids flow from an area of higher pressure to one of lower pressure.

Fluids flow from an area of higher pressure to one of lower pressure.

A client who has reached the stage of acceptance in the grieving process appears peaceful but demonstrates a lack of involvement with the environment. How should the nurse address this behavior?

Ignore the client’s behavior when possible.

Accept the behavior the client is exhibiting.

Explore the reality of the situation with the client.

Encourage participation within the client’s environment.

Accept the behavior the client is exhibiting.

Which physiologic changes may occur during the first trimester of pregnancy?

Fatigue

Increased libido

Morning sickness

Breast enlargement

Braxton Hicks contractions

Fatigue Morning sickness Breast enlargement **Fatigue, morning sickness, and breast enlargement are observed during the first trimester of pregnancy. Increased libido is observed during the second trimester of pregnancy. Braxton Hicks contractions are observed during the third trimester of pregnancy.

The registered nurse asks a client to rate his or her pain on a scale from 0 to 10, then instructs the nursing student to perform a physical assessment. Which assessments performed by the nursing student would be appropriate?

Palpating for tenderness

Observing nonverbal cues

Inspecting any areas of discomfort

Noticing if the pain localized or radiated

Noticing if the client gives nonverbal signs of pain

Palpating for tenderness Inspecting any areas of discomfort

A nurse is providing immediate postoperative care to a client who had a lung resection for a malignancy. The client has a closed chest tube drainage system connected to suction. Which assessment finding requires additional evaluation by the nurse?

A column of water 20 cm high in the suction control chamber

75 mL of bright red blood in the drainage collection chamber

An intact occlusive dressing at the insertion site

Constant bubbling in the water seal chamber

Constant bubbling in the water seal chamber **Constant bubbling in the water seal chamber is indicative of an air leak

Refusing to follow the prescribed treatment regimen, a client plans to leave the hospital against medical advice. What is it important for the nurse to inform the client of?

That the client is acting irresponsibly

That this action violates the hospital policy

That the client must obtain a new primary healthcare provider for future medical needs

That the client must accept full responsibility for possible undesirable outcomes

That the client must accept full responsibility for possible undesirable outcomes

The registered nurse is teaching the nursing student about the realms of family life. Which component does the registered nurse include while teaching about integrity processes?

Family rituals

Family relationships

Family life stressors and daily hassles

Family care takings and responsibilities

Family rituals **The family health system includes five realms, or processes, of family life. These realms are interactive, developmental, coping, integrity, and health. This approach is a method for family assessment used to determine areas of concern and strengths and to help develop an effective care plan. The component of integrity includes family rituals. Family relationships are a part of interactive processes. Family life stressors and daily hassles are considered components of coping processes. Health processes include family care takings and responsibilities.

A client presents with a shiny appearance of abdominal skin. The skin also has a taut appearance. Which condition may the client have?

Ascites

Cyanosis

Accidental injury

Bleeding disorder

Ascites

What should the nurse teach the parents about introducing a 6-month-old infant to solid foods?

The infant should be offered one new solid food at a time.

The infant may be offered fruit juices or fruit-flavored drinks.

The infant should be offered solid foods after the first birthday.

The infant should receive iron supplements in addition to solid foods.

The infant should be offered one new solid food at a time.

How does the World Health Organization (WHO) define "health"?

A condition when people are free of disease

A condition of life rather than pathological state

An actualization of inherent and acquired human potential

A state of complete physical, mental, and social well-being

A state of complete physical, mental, and social well-being

A pregnant woman in her second trimester arrives at the local health department, requesting a flu shot. The client states that she gets the flu vaccine every year and has never had an adverse reaction. What action should the nurse perform?

Do not administer the vaccine until checking with the healthcare provider.

Do not administer the vaccine due to pregnancy contraindication.

Administer the usual dose of the vaccine.

Administer half the usual dose of the vaccine.

Administer the usual dose of the vaccine.

A nurse is caring for a client for whom segmental postural drainage treatments are prescribed. The nurse should avoid scheduling the treatment at what time?

At bedtime

After a meal

One hour before a meal

One hour after awakening

After a meal **Productive coughing induced by postural drainage can cause nausea and vomiting. Because coughing must be encouraged after treatment, sleep will be postponed; however, as breathing is facilitated, sleep may become more restful. Approximately one hour before meals is a preferred time for postural drainage; the resulting cough and production of mucus will subside before eating. Upon awakening, mucous secretions are plentiful and tenacious; postural drainage at this time will be most beneficial.

The registered nurse is teaching a nursing student about the safety guidelines for nursing skills. Which statement by the student nurse indicates the need for further education?

"I should set up and prepare medications in distraction-free areas."

"I should advise the certified medical assistant to administer intravenous medication."

"I should be vigilant during the entire process of medication administration."

"I should identify each client using at least two identifiers before administrating medications."

"I should advise the certified medical assistant to administer intravenous medication."

A nurse is planning to provide self-care health information to several clients. Which client should the nurse anticipate will be most motivated to learn?

A 55-year-old client who had a mastectomy and is very anxious about her body image

An 18-year-old client who smokes cigarettes and is in denial about the dangers of smoking

A 56-year-old client who had a heart attack last week and is requesting information about exercise

A 47-year-old client who has a long-leg cast after sustaining a broken leg and is still experiencing severe pain

A 56-year-old client who had a heart attack last week and is requesting information about exercise

A client complains of rapid, involuntary movement of the eyes after a minor eye injury. A nurse assesses the client and finds that it is a disorder of the cranial nerves. Which condition does the nurse suspect?

Cataract

Glaucoma

Nystagmus

Strabismus

Nystagmus

The nurse at the well baby clinic is assessing the gross motor skills of a five-month-old infant. Which finding is a cause for concern?

The baby has a head lag when pulled to sit.

The baby can turn from the side to the back.

The baby can turn from the abdomen to the back.

The baby supports much of his own weight when he or she is pulled to stand.

The baby has a head lag when pulled to sit.

The nurse is caring for a client who had a hip replacement 2 days prior. After removing a bedpan from under the client, what is a priority nursing intervention?

Provide perineal care.

Turn and position the client.

Give a complete bed bath.

Document the bowel movement.

Provide perineal care.

Share This
Flashcard

More flashcards like this

NCLEX 10000 Integumentary Disorders

When assessing a client with partial-thickness burns over 60% of the body, which finding should the nurse report immediately? a) ...

Read more

NCLEX 300-NEURO

A client with amyotrophic lateral sclerosis (ALS) tells the nurse, "Sometimes I feel so frustrated. I can’t do anything without ...

Read more

NASM Flashcards

Which of the following is the process of getting oxygen from the environment to the tissues of the body? Diffusion ...

Read more

Unfinished tasks keep piling up?

Let us complete them for you. Quickly and professionally.

Check Price

Successful message
sending